You are on page 1of 50

Justin Sucgang

Statutory Construction
STATUTES Manner of referring:2
1. Public Act- passed by Philippine
Introduction Commission and Philippine Legislature
(1901-1935)
Law- a rule of conduct formulated and made obligatory 2. Commonwealth Act- passed during the
by legitimate power of the state. Commonwealth (1936-1946)
3. Republic Act- passed by Congress of the
Statute- an act of the legislature, as an organized body, Philippines (1946-1972;1987-present)
expressed in the form, and passed according to the 4. Batas Pambansa- passed by Batasang
procedure required to constitute it as part of the law Pambansa
of the land.1
Enacting statutes
Classification:
According to scope: Legislative power- authority to make laws, and to alter
1. Public- affects the public at large or the whole and repeal them
community
a. General- applies to the whole state *This is held by the people, in their original, sovereign
and operates upon all people or all and unlimited power. But they have
class; does not omit any subject or vested it in the Congress of the Philippines.3
place
b. Special- relates to a particular Procedures in passing a law:
persons or things or class or to a *Apart from the provisions in the Constitution
particular community
(Sec. 26 Par. 2, Art. VI)4, each house has its own
c. Local- confined to a specific place or detailed procedures embodied in their Rules (Sec.
community
16 Par. 3, Art. VI)5.
2. Private- applies to a specific person or subject
According to duration:
1. A bill is introduced by any member of the
1. Permanent- operation is not limited in
Congress, signed by the authors and filed with
duration but continues until repealed
the Secretary of the house. May introduce in
2. Temporary- duration is for a limited period; either house except appropriation, revenue or
ceases upon the happening of an tariff bills, bills authorizing increase of public
event for which it was passed debts, bills of local applications, and private
bills (Sec. 24 Art. VI).6
Other classifications:
2. First reading- Secretary reports the bill for first
According to application: reading:
1. Prospective 2. Retroactive - Reading the title and number of bill
According to operation:
- Referral to appropriate committee for
1. Declaratory study and recommendation
2. Curative - Committee may hold public hearings
3. Mandatory and submit its report and
4. Directory recommendation for Calendar for
5. Substantive second reading
6. Remedial 7. Penal According to form: 3. Second reading- bill shall be read in full with
1. Affirmative the proposed amendments by the committee
2. Negative

1 Presidential Decrees during martial law Congress of the Philippines which shall consist of a Senate and a House of
(Marcos) and Executive Orders during the Representatives, except to the extent reserved to the people by the
revolutionary government (Aquino) are also provision on initiative and referendum.
considered statutes since they were passed
by the president in the exercise of his 4 x x x (2) No bill passed by either House shall become a law unless it has
legislative powers. passed three readings on separate days, and printed copies thereof in its
2 Statutes are serially numbered and has a final form have been distributed to its Members three days before its
name passage, except when the President certifies to the necessity of its
immediate enactment to meet a public calamity or emergency. Upon the
3
Section 1 (Const). The legislative power last reading of a bill, no amendment thereto
shall be vested in the

JUSTIN SUCGANG | BLOCK 2 ATTY. WIGBERTO TANADA, JR.


-
Subject to debates, 3. When the vetoed bill is repassed by
pertinent motions and amendments Congress by two-thirds vote of all its
- After these, bill shall be voted upon members, voting separately.
4. Third reading- final vote by yeas and nay7
Enrolled bill- the bill as passed by Congress,
*After a house has approved their own version, it will authenticated by the House Speaker and Senate
be transmitted to the other house, which will follow President and approved by the President Rule: The text
the same procedures. If without amendments, the bill of the act as passed and approved is deemed
is passed by Congress and is submitted to the importing absolute verity and is binding on the
President. If there are courts. If there has been any mistake in the
printing
shall be allowed, and the vote thereon shall be taken immediately of the bill before it was certified, the remedy is
thereafter, and the yeas and nays entered in the Journal. x x x by amendment by enacting a curative
legislation and not a judicial decree.7
5 x x x (3) Each House may determine the rules of its proceedings, x x x
Presumption: It carries on its face a solemn
6
assurance by the legislative and executive
All appropriation, revenue or tariff bills, bills authorizing increase of the
public debt, bills of local application, and private bills, shall originate departments of the government that it was
exclusively in the House of Representatives, but the Senate may propose or passed by the assembly. Hence, the respect
concur with amendments.
due to coequal and independent departments
7
requires the judiciary to act upon that
If there is a presidential certification, the requirement of three readings on
separate days, and printed copies in final form may be dispensed with assurance and to accept all bills duly
(Tolentino v. Secretary of Finance). authenticated.8

amendments, there will be a Conference Journal Entry- a requirement by the Constitution to


Committee each house of the Congress (Sec. 16 Par.4 Art.
VI)9
5. Conference Committee- this is where the - conclusive with respect to matters that are
differences will be settled. The amendments required by the Constitution to be recorded
introduced in this level will have to be therein
approved by both Houses for passage.5 - based from considerations of public policy
6. Approval and authentication - signing of the
Senate President and the House Speaker and *Enrolled bill v. Journal Entry
their respective secretaries -In case of conflict, the enrolled bill should prevail,
(Enrolled Bill) particularly with respect to matters not expressly
7. Submission to the President:6 A bill is passed in required to be entered into the legislative record.10
three ways: Exception: When the Speaker and Senate
1. When the president signs it President withdraw their respective signatures
2. When the president does not sign nor from the signed bill where there is serious and
communicate his veto of the bill within 30 substantial discrepancy between the text of
days after his receipt the bill as deliberated and shown by the
journal

5 There are instances where the version of the objections, to the other House by which it shall likewise be
the conference committee is entirely reconsidered, and if approved by two-thirds of all the Members of that
different from those of the two Houses- for House, it shall become a law. In all such cases, the votes of each House shall
it may deal generally with the subject matter be determined by yeas or nays, and the names of the
or precisely to the differences, or even Members voting for or against shall be entered in its Journal. The President
introduce a new provision. However, this is shall communicate his veto of any bill to the House where it originated
still valid for the powers of said committee within thirty days after the date of receipt thereof, otherwise, it shall
are broad. That is why some political become a law as if he had signed it.
scientists call this the Third House
(Philippine Judges Association v. Prado). 7 Casco Phil. Chemical Co., Inc. v. Gimenez

6 Section 27. (1) Every bill passed by the 8


Morales v. Subido
Congress shall, before it becomes a law, be
presented to the President. If he approves 9
x x x(4) Each House shall keep a Journal of its proceedings, and from time
the same he shall sign it; otherwise, he shall to time publish the same, excepting such parts as may, in its judgment,
veto it and return the same with his affect national security; and the yeas and nays on any question shall, at the
objections to the House where it originated, request of one-fifth of the Members present, be entered in the Journal.
which shall enter the objections at large in Each House shall also keep a Record of its proceedings. x x x
its Journal and proceed to reconsider it. If,
after such reconsideration, two-thirds of all 10
Morales v. Subido
the Members of such House shall agree to
pass the bill, it shall be sent, together with
JUSTIN SUCGANG | BLOCK 2 ATTY. WIGBERTO TANADA, JR.
and that of the enrolled bill.11 powers in me vested by the Constitution,
do hereby decree...”

Parts of a statute 4. Purview/Body of the Statute


- What the law is all about
1. Title - Should embrace one subject matter
- Bill must embrace only one subject - The provisions, although different and
expressed in its title12 diverse, must be allied and germane to the
- But must not be an index to, or be an subject and purpose of the bill -
exhaustive catalogue of the body of the act Usually divided into sections
as to cover every single detail (numbered and contains a single
proposition)
- It is enough that the title indicates the
general subject, and reasonably covers all - Usually includes a short title, policy,
provisions of the act and not calculated to definition, administrative sections, sections
mislead the public Reasons: prescribing standards of conduct, imposing
sanctions for violations of its provisions,
- Prevent hodgepodge or log-rolling
transitory
legislation
provisions
- Prevent fraud and surprise through
introduction of provisions not germane to
the statute (which have not received 5. Separability Clause
notice, action and study of the legislators) - States that if any provision is declared
- Language that must be sufficient to notify invalid, the remainder shall not be affected
the legislators and the public Presumption: Legislature intended a statute to
be effective as a whole and would not have
- Title serves as guide to ascertain legislative
passed it had it foreseen that some part of it is
intent How construed:
invalid. Exception: Where provisions cannot
- Should be liberally construed
stand alone as to those left, after the void part,
- Not be given technical interpretation nor is not complete and workable
narrowly construed to cripple or impede
the power of the legislature
6. Repealing Clause
- If there is doubt, it should be resolved in
favour of the one title-one subject
(presumption of constitutionality) 7. Effectivity Clause
- When the law takes effect
2. Preamble - Usually 15 days from the publication in the
Official Gazette13 or in a
- Prefatory statement or explanation
(finding of facts, reciting the purpose, newspaper of general circulation14
reason, or occasion for making the law).
Hence, important role in construction Kinds of statutes
- Usually found after the enacting clause and
before the body in presidential decrees 1. Legislative acts
and executive orders
- Legislature seldom puts this because the 2. Presidential issuances- those which the
reason for the law is contained in its president issued in the exercise of his
explanatory note ordinance power (Chapter 2, Book III, AC)
a. Sec. 2. Executive Orders. - Acts of
3. Enacting Clause the President providing for rules of a
- Part written immediately after general or permanent character in
the title implementation or execution of
- States the authority by which constitutional or statutory powers
the act is enacted shall be promulgated in executive
- Contains the phrases, “Be it enacted by...” orders.
or “Now, therefore, I... by virtue of the

11 Astorga v. Villegas 13Art. 2 (CC). Laws shall take effect after fifteen days following the
completion of their publication in the Official Gazette, unless it is otherwise
12Art. VI Sec. 26 (1). Every bill passed by the provided. This Code shall take effect one year after such publication.
Congress shall embrace only one subject
14Art. 18 (AC) Laws shall take effect after fifteen (15) days following the
which shall be expressed in the title thereof.
xxx completion of their publication in the Official Gazette or in a newspaper of
general circulation, unless it is otherwise provided
JUSTIN SUCGANG | BLOCK 2 ATTY. WIGBERTO TANADA, JR.
b. Sec. 3. Administrative *The rule-making power of a public
Orders. - Acts of the President which administrative agency is a delegated legislative
relate to particular aspect of power. It may not use its power to bridge the
governmental operations in pursuance authority to enlarge its power beyond the
of his duties as administrative head scope intended.
shall be promulgated in administrative
orders. *The law passed by the legislature should be:
c. Sec. 4. Proclamations. - Acts of the (1) complete in itself, and (2) should fix a
President fixing a date or declaring a standard, in order for an administrative agency
status or condition of public moment to fill in the details in the execution,
or interest, upon the existence of enforcement and administration of said law.
which the operation of a specific law or
regulation is made to depend, shall be - Administrative rule- promulgating a new
promulgated in proclamations which law with force and effect of a valid law.
shall have the force of an executive - Administrative interpretationsrendering an
order. opinion or giving a
d. Sec. 5. Memorandum Orders. - Acts of statement of policy
the President on matters of
administrative detail or of subordinate *The rules are binding upon the Courts, but the
or temporary interest which only interpretation is not.
concern a particular officer or office of
the Government shall be embodied in 4. Supreme Court rule-making power- the
memorandum orders. Constitution granted the Supreme
e. Sec. 6. Memorandum Circulars. - Acts Court to promulgate its own rules
of the President on matters relating to (Sec. 5 Par. 5, Art. VIII)15
internal administration, which the *Usually procedural only for only the legislative
President desires to bring to the department may create substantive laws.
attention of all or some of the
departments, agencies, bureaus or
5. Local government units- power to enact
offices of the Government, for
ordinances16
information or compliance, shall be
embodied in memorandum circulars.
Requirements of validity:17
f. Sec. 7. General or Special Orders.- Acts
and commands of the President in his 1. Not contravene the Constitution or any
capacity as Commander-in-Chief of the statute
Armed Forces of the Philippines shall 2. Not be unfair or oppressive
be issued as general or special orders. 3. Not be partial or discriminatory
4. Not prohibit but may regulate trade
3. Administrative rules and regulations- issued 5. General and consistent with public policy
by administrative or executive offices in 6. Not unreasonable
accordance with, and as authorized by, law
have the force and effect of law or partake the Barangay/Municipal/City/Provincial
nature of the statute Requirement for validity: ordinances
1. Provisions should be germane to the - Passed by majority vote of council
objects and purpose of the law. members
2. Not in contradiction with, but conform to, - Submitted to the mayor (for
the standards that the law prescribes municipal/city), if unacted for 10 days,
3. They be for the sole purpose of carrying considered passed; governor (for
into effect the general province), if unacted for 15 days,
provisions of the law considered passed
- Vetoed ordinance may be passed by a
vote of 2/3 of council members

15x x x (5) Promulgate rules concerning the courts of the same grade, and shall not diminish, increase, or modify
protection and enforcement of substantive rights. Rules of procedure of special courts and quasi-judicial
constitutional rights, pleading, practice, and bodies shall remain effective unless disapproved by the Supreme Court.
procedure in all courts, the admission to the
practice of law, the integrated bar, and legal 16 Local Government Code of 1991
assistance to the under-privileged. Such
rules shall provide a simplified and 17 Lagcao v. Labra
inexpensive procedure for the speedy
disposition of cases, shall be uniform for all
JUSTIN SUCGANG | BLOCK 2 ATTY. WIGBERTO TANADA, JR.
- Subject to review by sustained or will sustain direct injury
sangguniang bayan o
panglunsod/sangguniang panlalawigan 1. Citizen standing- suffered some actual or
- If no action in 30 days, ordinance will threatened injury as a result of the
be presumed consistent with laws allegedly illegal conduct of the government
(Tanada v. Tuvera)
Validity of statutes 2. Taxpayer’s standing- public funds have
been disbursed in alleged contravention of
Presumption of validity the law or Constitution (ITF v. COMELEC)
Basis: Before the legislature passes a bill, it has decided Basis: Expenditure of public funds by an
the measure to be constitutional. And when the officer of the state for the purpose of
President approves the bill, he has been convinced of administering an unconstitutional act
its validity. constitutes a misapplication of such funds
Effect: To justify the nullification of the law, there must 3. Legislator’s standing- when their powers
be clear and equivocal breach of the Constitution, not are impaired; questioning the validity of a
a doubtful and argumentative implication. presidential veto (Ople v. Torres)
4. Transcendental significance- the
*The final authority to declare unconstitutionality is Court has adopted a liberal attitude
the Supreme Court sitting en banc (Sec. 4 Par. 2, (brushing aside technicalities of procedure)
Art. VIII).18 on standing where the petitioner has
shown that an issue has paramount
Judicial Power19 importance to the public (Chavez v. PEA
and AMARI)
1. Judicial power includes the duty of the courts
of justice to settle actual controversies
involving rights which are legally demandable Raised at the earliest possible time
and enforceable - Question must be raised in the pleading,
2. Determine whether or not there has been a complaint, information or petition by the
grave abuse of discretion amounting to lack or plaintiff or petitioner or in the answer by
excess of jurisdiction on the part of any branch defendant or respondent.
or instrumentality of the Government.
Exception:
Requisites: 1. Can be raised in a motion for reconsideration if
1. Actual case and controversy the statute sought to be invalidated was not in
existence when the complaint was filed.20
2. Locus standi
3. Raised at the earliest possible time 2. At any stage of the proceedings in a
4. Lis mota of the case criminal case21
3. Where the determination of the question is
Actual case and controversy necessary to the decision for civil cases22
- There must be a justiciable controversy (can be 4. Where it involves jurisdiction of the court
decided on grounds recognized by the law) below23
Exceptions: Political questions- issues dependent
upon wisdom not the legality of
Lis mota of the case
the law
- The Court may not pass upon the validity of a
statute if it can decide the case on some other
Locus standi grounds
- Legal standing to sue - If the only issue is the a constitutional question
- The person has substantial interest in the case which is unavoidable, the Court should
such that the party has confront the question and decide the case on
the merits

18 x x x(2) All cases involving the concurrence of a majority of the Members who actually took part in the
constitutionality of a treaty, international or deliberations on the issues in the case and voted thereon.
executive agreement, or law, which shall be
heard by the Supreme Court en banc, and 19 Sec. 1, Art. VIII (Const)
all other cases which under the Rules of 20
Alonso v. PNB
Court are required to be heard en banc,
including those involving the 21
San Miguel Brewery v. Magno
constitutionality, application, or operation
of presidential decrees, proclamations, 22 Id
orders, instructions, ordinances, and other
23 26 Id
regulations, shall be decided with the
JUSTIN SUCGANG | BLOCK 2 ATTY. WIGBERTO TANADA, JR.
it and determines the rights of the parties
Test of constitutionality just as if it had no existence
1. Must not contravene the Constitution or any - The decision affects the parties only
statute - No judgment against the statute: opinion of the
2. Must be general and consistent with public court may act as a precedent, but it does not strike
policy out, repeal, supersede, revoke, or
3. Must not be unfair or oppressive annul it
4. Must not be partial or discriminatory
5. Must not be unreasonable Exception: Invalidity Due to Change of Conditions
6. Must not prohibit but may regulate trade Inapplicable to a statute that is declared invalid
because of the change of circumstances affecting its
Grounds for nullification validity (emergency laws). The declaration of their
1. Not within the legislative power to enact nullity should be applied prospectively, and affect only
the parties involved in the case.
2. When it allows something to be done which
the fundamental law condemns or prohibits
Partial Invalidity
3. Vagueness- lacking comprehensible standards
General rule: Where the part of the statute is void
that men of common intelligence must
as repugnant to the Constitution, while another
necessarily guess at its meaning and differ in its
part is valid, the valid portion, if separable from
application Basis:
the invalid, may stand and be enforced
- Violates due process for failure to
(separability clause). Reason: Legislature intended
accord the people fair notice of what
a statute to be effective as a whole and would not
conduct to avoid.
have passed it had it foreseen that some part of it
- Leaves law enforcers unbridled is invalid
discretion in carrying out its provisions Exception: When the parts of the statute are so
and becomes an mutually dependent and connected, as conditions,
arbitrary flexing of the
considerations, inducements, or compensations for
government muscle each other, as to warrant a belief that the
legislature intended them as a whole, the nullity of
Effects of unconstitutionality one part will vitiate the rest.
General rule: An unconstitutional act is not a law,
confers no right, imposes no duties, affords no Effectivity of statutes
protection, creates no office; in legal contemplation,
inoperative as though it had never been passed.
General rule: Statutes continue to be in force until
changed or repealed by the legislature.
*This is also known as the orthodox view. Not only the
parties but all persons are bound by declaration of
*Not changed by change of sovereignty, conquest or
nullity; no one may invoke it nor may the courts be
colonization.
permitted to apply it
subsequently
1. For statutes proper, usually 15 days from the
publication in the Official Gazette28 or in a
*The orthodox view is expressed in Art. 7 (CC).24 newspaper of general circulation,29 unless
However, it is not always the case that a law is provided otherwise. But the ruling may seem in
constitutionally faulty per se (i.e., may be valid in its contradiction with the PVB Employees Union v.
general import but invalid in its application to certain Vega, the phrase “unless otherwise provided”
situations). A judicial declaration of nullity may not provided an exception as to the date of
necessarily obliterate all the effects and consequences effectivity of a statute.
of a void act occurring prior to such declaration. A
2. For issuances, rules and regulations, there
situation that is fait accompli
should be a publication30 and filing in the the
may no longer be open for
UP Law Center.31
inquiry, let alone to be
unsettled by a subsequent declaration of nullity of a
statute.

Modern view
- The court does not annul or repeal statute if it
violates Constitution; it simply refuses to recognize

24Art. 7 (CC). When the court declares a law former shall be void and the latter shall govern x x x
to be inconsistent with the Constitution, the
JUSTIN SUCGANG | BLOCK 2 ATTY. WIGBERTO TANADA, JR.
3. For ordinances, Sec. 59 of the
Local Government Code shall apply.25

28Art.2 (CC). Laws shall take effect after fifteen days following the
completion of their publication in the Official Gazette, unless it is otherwise
provided. This Code shall take effect one year after such publication.
29
Sec. 18 (AC) Laws shall take effect after fifteen (15) days following the
completion of their publication in the Official Gazette or in a newspaper of
general circulation, unless it is otherwise provided

30 Id

31 Sec. 3, Book VII (AC). Filing. - (1) Every agency shall file with the University
of the Philippines Law Center three (3) certified copies of every rule
adopted by it. Rules in force on the date of effectivity of this Code which
are not filed within three (3) months from that date shall not thereafter
be the basis of any sanction against any party or persons.

Manner of computing time:


When the laws speak of years, months, days or
nights, it shall be understood that years are of three
hundred sixty-five days each; months, of thirty
days; days, of twenty-four hours; and nights from
sunset to sunrise.

If months are designated by their name, they shall


be computed by the number of days which they
respectively have.
circulation within the city: Provided, That in the absence thereof the
ordinance or resolution shall be published in any newspaper of general
In computing a period, the first day shall be excluded, circulation.

and the last day included.33 33

Art. 13 (CC)

Exception: Computation of the prescription of a


crime. If the last fell on a Sunday or a legal holiday,
a charge cannot be filed on the next working day
since it shall be considered prescribed.
CONSTRUCTION AND INTERPRETATION warranted conclusions not always included in direct
expressions, or determining the application of words
-Art or process of discovering and expounding the to facts in litigation.26
meaning and intention of the authors of the law,
where that intention is rendered doubtful by reasons Purpose: Ascertaining the true intent of the legislature.
of ambiguity in its language or the fact that the given Assumption: The legislature enacts a law with the end
case is not explicitly provided for in the law. in view that it will, in cases of doubt, be construed in
Basis: Because of infirmities of language and the accordance with the settled principles of
limited scope in legislative drafting. interpretation.

Interpretation- Art of finding the true meaning and *Rules of statutory construction are used to ascertain
sense of any form of words Construction- Drawing legislative intent. But they are not rules of law but

25 Effectivity of Ordinances or Resolutions. government unit concerned not later than five (5) days after approval
(a) Unless otherwise stated in the ordinance thereof.
or the resolution approving the local The text of the ordinance or resolution shall be disseminated and posted in
development plan and public investment Filipino or English and in the language or dialect understood by the majority
program, the same shall take effect after of the people in the local government unit concerned, and the secretary to
ten (10) days from the date a copy thereof is the sanggunian shall record such fact in a book kept for the purpose, stating
posted in a bulletin board at the entrance of the dates of approval and posting.
the provincial capitol or city, municipal, or (c) The gist of all ordinances with penal sanctions shall be published in a
barangay hall, as the case may be, and in at newspaper of general circulation within the province where the local
least two (2) other conspicuous places in the legislative body concerned belongs. In the absence of any newspaper of
local government unit concerned. (b) The general circulation within the province, posting of such ordinances shall be
secretary to the sanggunian concerned shall made in all municipalities and cities of the province where the sanggunian
cause the posting of an ordinance or of origin is situated. (d) In the case of highly urbanized cities, the main
resolution in the bulletin board at the features of the ordinance or resolution duly enacted or adopted shall, in
entrance of the provincial capitol and the addition to being posted, be published once in a local newspaper of general
26
city, municipal, or barangay hall in at least Conclusions which are in the spirit, though not within the letter of the
two (2) conspicuous places in the local text.
JUSTIN SUCGANG | BLOCK 2 ATTY. WIGBERTO TANADA, JR.
mere axioms of experience, hence, not When to construe
binding nor controlling on the courts.
Condition sine qua non: Before the court may use its
Legislative intent- is the essence of the law. It is the power to construe, there must be ambiguity in the
spirit which gives life to legislative enactment language of the statute. For where there is no
Legislative purpose- reason why a particular statute ambiguity, there is no room for construction, only for
was enacted Legislative meaning- what the law, by its application.
language means
Ambiguity- condition of admitting two or more
*The primary source of legislative intent should be the meanings, of being understood in more than one way,
statute itself. or of referring to two or more things at the same time.
*susceptibility to more than one interpretation
Power to construe
- The duty and power to interpret or construe a More application, less construction
statute or the Constitution belongs to the judiciary - The first and fundamental duty of the court is
(Sec. 4 Par. 2, Art. VIII). to apply the law.
- It is the Court which has the final say as to what the - Construction comes only after there has been a
law means. showing of ambiguity, hence,
application is impossible
Limitations: The Court can only construe an
applicable law in controversies which are ripe for Verba legis (Plain meaning rule)
judicial resolution. Neither moot nor academic - Where the statute is clear, plain and free from
(purpose has become stale or where no practical ambiguity, it must be given its literal meaning
relief can be granted or which can have no practical and applied without interpretation.
effect). Exception: If the issue is “capable of - Where the law speaks in clear and categorical
repetition yet evading review” especially where language, there is no room for interpretation.
public interest requires its resolutions.27 There is only room for application.29
- Law cannot be changed under the guise of
*The legislature has no power to overrule the
interpretation.30 Maxim: index animi sermo
interpretation made by the Court.28
est (speech is the index of intention)
Presumption: The words employed by the
Meaning: The legislature cannot, by law or resolution, legislature in a statute correctly express its
modify or annul the judicial construction without intention or will and preclude the court from
modifying or repealing the very statute which has been construing it differently.
subject of construction.
Silva v. Cabrera
Exceptions: Cabrera wanted to construct an ice plant even if there is an
1. When the Supreme Court reverses itself existing one already owned by petitioner. The case was
2. By amending the Constitution brought to the Public Service Commission, and the agency
3. By enacting a new statute appointed Atty. Espellera to hear the case and receive
evidence. The PSC rendered a decision allowing respondent
to operate.
Held: Where the law is clear, there is only room for
Endencia v. David application. Although the law granted the PSC to appoint an
After a judicial declaration interpreting a constitutional attorney to take testimony of the witnesses if residing
provision that taxing is a form of diminution of salary, distant from Manila, the law is clear that only the
Congress enacted a law to include justices and members of commission can hear, receive evidence, and render a
the judicial body in the scope of taxing power of the decision.
government. Held: Congress cannot, by law, modify an
interpretation of the Constitution
Quijano v. DBP
made by the court. Including the
Petitioner, a veteran, obtained a loan from DBP. He wanted
judicial officers to the scope of
to pay the subsisting obligation through the backpay
taxation is a form of interpretation of the Constitutional
certificate.
provision against diminution of salaries.
Held: The law is clear, backpay certificate may only be used
to pay for the loan if the subsisting obligation was obtained

27 Pimentel v. Ermita meaning by interpretation and applied it in a decision, this would surely
cause confusion.” (Endencia v. David)
28 29 Cebu Portland Cement v. Municipality of Naga
“If the legislature may declare what a law
means, or what a specific portion of the
30 Crisologo v. Macadaeg
Constitution means, especially after the
courts have in actual case ascertained its
JUSTIN SUCGANG | BLOCK 2 ATTY. WIGBERTO TANADA, JR.
prior to the passage of RA 897. A subsisting People v. Nazario
obligation may only be considered after receipt of the loan, Nazario, a lessee of land used as fishpond, was taxed under a
not on the day of awarding of application (since, there was municipal ordinance. He contended that said ordinance was
still no obligation at that time). vague.
Held: The law is not vague. The word “owner” should be
Commissioner of Internal Revenue v. Limpan Investment construed as to include petitioner. Since the land is owned
The BIR found that respondent had tax deficiencies for more by the government, for he is a mere lessee, it cannot be said
than two years. They brought the case to the Court of Tax that the tax should shouldered by the latter for being the
Appeals where it held that the interest and surcharge should owner. Petitioner, being the operator of the pond, manager
start from period where the decision became final. of the workers, and the one who gets profit from it should
Held: The National Internal Revenue Code clearly stated that be responsible for the tax burden.
in computing the interest and surcharge of the tax
deficiency, it should start from the date of notice and Ursua v. CA
demand. The CTA, therefore, made an error where it fixed a Usua was charged with a case for allegedly using an alias in
new date. claiming a complaint. Since the messenger of his legal
counsel was attending some personal matters, petitioner
People v. Mapa asked for the permission to use the messenger’s name.
Mapa claimed that he was a secret agent of Governor Leviste Later, it was found out that he signed using that
when he was charged for illegal possession of unlicensed messenger’s name. Held: The law penalizing the use of
firearms. aliases should not be construed literally. It should be
Held: Although it was earlier held that secret agents may be interpreted in light of the purpose it serves and the evils it
exempted from possession of firearms since they do the seeks to prevent. Ursua’s use of a fictitious name in a single
work of peace officers, the Revised Administrative Code instance without any sign that he intended to be known by
clearly enumerated those who may be exempted, and secret this name does not fall within the provision of CA 142.
agents were not included.
Supreme Court rulings part of the legal system
Chartered Bank Employees v. Ople Employees of Chartered
Bank complained that they were not paid for the 10-day
Judicial decisions applying or interpreting the laws or
holiday. Employer said that a circular made by Sec. Ople
the Constitution shall form part of the legal system of
clarified that regular employees are presumed to be paid for
the whole year, thereby also counting holidays. Held: The the Philippines.31
Labor Code clearly stated that holidays should be paid by the Maxim: legis interpretato legis vim obtinet (the
employer to regular employees, hence, the circular is invalid. authoritative interpretation of a statute acquires the
Moreover, it was also found out that the computation for force of the law by becoming a part thereof)
the salaries did not include said holidays, hence, the - As of the date of the date of enactment
employer is really liable for said payment. - Interpretation establishes the
contemporaneous legislative intent
Melendres v. COMELEC
Melendres lost to Concepcion in a barangay election. He
stare decisis et non quieta movere
filed an election protest to an MTC. Concepcion filed a
petition to the COMELEC and moved to dismiss it upon - When the Supreme Court has once laid down
knowing that no docket fee was paid. Hence, petitioner a principle of law as applicable to a certain
questioned the jurisdiction of the COMELEC. Held: Payment state of facts, it will adhere to that principle
of docket vests jurisdiction to a court (Gatchalian v. CA). and apply to all future cases where the facts
Non-payment of this means no protest must be given due are substantially the same
course. The COMELEC is right when it dismissed it since its - Assures certainty and stability in the legal
Rules of Procedure provided that protests should be filed 10 system
days after the proclamation; the payment was done 25 days
after said proclamation.
Perfecto v. Meer Held:

Ratio legis (Spirit of the law)


Endencia v. David
- Moving away from the literal interpretation Following the ruling in Perfecto v. Meer that a tax on SC
Maxims: ratio legis et anima justices and other officials of the judiciary is a diminution of
legis (the reason of the law is their salary, contrary to the Constitution, Congress enacted a
its soul); ratione cessat lex et law including subjecting them to income tax and declaring
cessat lex (when the reason of the law ceases, the that it is not a diminution of their salary.
law ceases) Presumption: The letter of the law is Held: Congress cannot overrule, in a legislative action, a
its body; the spirit (reason), the soul; and the construction of a constitutional provision made by the Court.
construction of the former should never be so In declaring that “subjecting court officials to income tax is
not a diminution of their salary,” they are invading the
rigid and technical as to destroy the latter.
province of the Court to interpret laws.

31Art. 8 (CC). Judicial decisions applying or shall form a part of the legal system of the Philippines.
interpreting the laws or the Constitution
JUSTIN SUCGANG | BLOCK 2 ATTY. WIGBERTO TANADA, JR.
Held: Although the then prevailing ruling was
People v. Canton People v. Mapa, where secret agents are not anymore
The metal detector beeped when Susan Canton passed considered as peace officer, therefore, not exempted from
through it. Initially she said that it was only money, but the permit to carry firearms, it was non-existent when Licera
lady frisker called her superior. She was brought inside a rest was charged. The then ruling was People v. Macarandang
room and was frisked. Several grams of shabu was found in where the Court interpreted secret agents as peace officers.
her abdominal, genital, and thigh areas. Hence, the latter should apply to this case since Licera relied
Held: Although the Constitution grants the right to privacy, on it in good faith.
the Terry search conducted to her is valid. The arrest
without a warrant was justified since it was only carried out *Only Supreme Court sitting en banc may modify or
upon discovery and recovery of shabu in her as ruled in abandon an established principle of law, not any
People v. Johnson. division (Sec. 4 Par. 3, Art. VIII)32 Exception: If the Court
has stated in a decision not a principle of law but a
DAR v. Sutton mere obiter dictum (a thing said in passing), a division
Respondent inherited a land which they devoted for cattle may validly reject or disregard it.
breeding. Initially they availed of voluntary offer to sell
because of incentives, but withdrew it after the Court
*The Court has the duty to formulate guiding and
promulgated the doctrine exempting agricultural lands used
for poultry and livestock breeding in Luz Farms v. DAR controlling constitutional principles, precepts or
Secretary. Petitioner only granted partial exemption on the doctrines. The power to issue guidelines is not judicial
basis of an administrative order issued. legislation, the Court merely defines what the law is.
Held: The ruling of the Court should be enforced. In its Examples:
interpretation in Luz Farm, it clearly declared the exemption 1. Partylist: Ang Bagong Bayani-OFW Labor Party
of said lands used in poultry and livestock. The v. COMELEC
administrative order providing for a 1:1 ratio is 2. Psychological incapacity: Republic v. CA and
unconstitutional. Although the DAR may issue orders and
Molina
regulation, these are subject to judicial review.
3. Anti-subversion: People v. Ferrer
*The interpretation of a statute remains to be part of 4. Rights when arrested: Morales v. Enrile
the legal system until the Supreme Court overrules it
and the new doctrine overruling the old is applied Floresca v. Philex Mining
Several workers of respondent died when the mine
prospectively in favour of parties who relied on it in
collapsed. The petitioners asked for compensation which
good faith.
was given. After learning that there was negligence on the
part of the company, petitioners sued and asked for
*Judicial ruling cannot be given retroactive effect if to damages under the Civil Code.
do so will impair vested rights and the parties relied on Held: There was no judicial legislation when the
it in good faith. Supreme Court held that petitioners can still claim
Maxim: lex prospicit, non respicit (the law looks damages in civil courts even if the Workmen’s
forward, not backward) Compensation Act expressed that its Commission shall

Columbia Pictures v. CA have exclusive original jurisdiction. It just merely gives


After being granted a petition for a search warrant in an
alleged place where pirated movies were produced, the CA life to the law.
dismissed the case when the SC promulgated a new doctrine
requiring the presentation of the master copy.
Held: Although court interpretations are deemed part of the
law during its enactment, it could not be applied *The court should interpret the law as a whole
retrospectively especially when it introduces a new ruling
and vested rights were accrued before (production of master
copy). Acop v. Guingona Two police officers testified that
the Kuratong Baleleng controversy was in fact a rub-out
not a shoot-out. Sens. Drilon and Roco placed them under
People v. Licera
the witness protection program.
Defendant was charged with illegal
Held: Although in Sec. 3 clearly said that only the
possession of firearms. He
relatives of police officers and not themselves are
contended that he is exempted
qualified for said protection, Sec. 4 provided that the
from this rule saying that he was a secret agent employed by
Senate and the House may also give this protection to
Gov. Leviste of Batangas.

any person. The statute must be interpreted as a

32
(3) Cases or matters heard by a division and in no case without the concurrence of at least three of such Members.
shall be decided or resolved with the When the required number is not obtained, the case shall be decided en
concurrence of a majority of the Members banc: Provided, that no doctrine or principle of law laid down by the court in
who actually took part in the deliberations a decision rendered en banc or in division may be modified or reversed
on the issues in the case and voted thereon, except by the court sitting en banc.
JUSTIN SUCGANG | BLOCK 2 ATTY. WIGBERTO TANADA, JR.
whole, giving life to all its parts, hence,
Sec. 4 should be seen as an exception to the provision in
Sec. 3.

Limitations on power to construe

1. Courts may not, under the guise of


interpretation, enlarge the scope of a
statute and include therein situations not
provided nor intended by lawmakers.33
2. Neither should courts construe statutes
which are perfectly vague for it is
repugnant to the Constitution (see test of

Constitutionality)
3. Courts do not pass upon questions of
wisdom, justice or expediency of
legislation. For any shortcoming of a
statute is for the legislative alone to
correct by appropriate enactment.34

33“Courts are not authorized to insert into (People v. Garcia)


the law what they think should be 34
Lacson v. Roque
in it or to supply what they think the
legislature would have supplied if its
attention had been called to the omission.”

JUSTIN SUCGANG | BLOCK 2 ATTY. WIGBERTO TANADA, JR.


-
AIDS TO CONSTUCTION - Found only in presidential decrees issued by
the president in the exercise of his legislative
*Where the meaning of the statute is ambiguous, the power since Congress usually
courts may avail of itself all legitimate aids to include an explanatory note
construction to ascertain the true intent of the
legislature. People v. Purisima
PD 9 was promulgated making the carrying of bladed
Intrinsic- found in the printed page of the statute weapons criminal. Several individuals in different provinces
were caught and charged for this. Held: The Court, in using
Extrinsic- extraneous facts and circumstances outside
the “whereas” clauses, said that in order for this to be a
the printed page
crime, it should be in furtherance to rebellion, sedition and
coup d’etat. Clearly, the acts should be within the intent of
Intrinsic Aids PD 1081 and GO 6 and 7.

1. Title Context of the whole text


2. Preamble - Best source to ascertain legislative intent is the
3. Context of the whole text statute itself- words, phrases, sentences,
4. Punctuation marks sections, clauses, provisions
5. Capitalization of letters - Taken as a whole and in relation to another;
6. Headnotes or epigraphs not from an isolated part or particular provision
7. Language/ lingual text
PRC v. De Guzman
Medicine students from Fatima University got unusually high
Title scores in their board exam. This prompted the PRC to
- It may indicate the legislative intent to extend conduct an investigation. Afterwards, they did not allow said
or restrict the scope of the law examinees to take their oath.
- Carries more weight because of the Held: The law should be interpreted as a whole. Although it
constitutional requirement of one bill-one ordered the PRC to allow students, upon completion of all
subject expressed in the title35 requirements to take oath in Sec. 22, the succeeding
provision qualified this by saying that it should be done upon
- An indispensable part of a statute for what may satisfactorily completing everything. With the use of a
inadequately be omitted in the text may be dictionary, the Court said that they should be capable of
supplied by its title (City of Baguio v. Marcos). dispelling doubt or ignorance.
Exception: When the text of the statute is
clear and free from doubt Basbacio v. Office of the Secretary of Justice
Petitioner was accused, together with his son-in-law, of
Ebarle v. Sucaldito murder and two counts of frustrated murder of a family
Gov. Ebarle sought to dismiss several criminal charges of whom they have land disputes. However, he was acquitted
graft and corruption due to non-compliance to EO 264 by the CA for insufficiency of evidence proving conspiracy.
outlining the procedures for complaints against government Hence, he petitioned for payment of compensation for being
officials with commission of irregularities. Held: The title of unjustly accused to the Secretary of Justice but was denied
the statute clearly talks about “commission of irregularities,” Held: The term “unjustly” should be construed in terms of
meaning to say, administrative complaints. Hence, this is not the manner of conviction and not on his acquittal. There
applicable to criminal procedures. While both may be should be malice in the accusation which results to unjust
considered crimes, the EO is rather specific about this. imprisonment and conviction. Agreeing on the contention of
the accused renders the provision with this qualifying word
Preamble ineffective.
Part written immediately after the title-
Garcia v. SSS
“Whereas” clauses
Garcia, the remaining member of the Board of Directors of
- Although not an essential part Impact Corporation, was charged for nonremittance of SSS
of the statute, it is important if dues.
there is ambiguity in the Held: As the remaining director, she is liable for payment,
meaning since it states the purpose, reason or not only of penalties, but also of the amount unremitted.
justification for the enactment of the law Although there is a provision, when accepted literally will
- Key of the statute- holds the purpose to be mean that liability pertains only to penalties, the law should
achieved, mischief to be remedied, and object
to be accomplished (legislative intent)

35Art. VI Sec. 26(1) Every bill passed by the which shall be expressed in the title thereof. x x x
Congress shall embrace only one subject

JUSTIN SUCGANG | BLOCK 2 ATTY. WIGBERTO TANADA, JR.


-
be interpreted as a whole. The 3% penalty, under Sec.22(a) references Exception: When the text of a
automatically attaches itself to the amount unremitted.36 statute is clear

NAPOLCOM v. De Guzman Language


With the passage of a new law, several Philippine - The original language when a statute is
Constabulary members received from petitioner a notice of officially promulgated will prevail
retirement for reaching the age of 56. They contended that
this age of retirement cannot be applied to them since the
- Revised Penal Code is Spanish; Judiciary Act is
PC is part of the Integrated National Police whose English
mandatory age of retirement has been set to 60. General rule: The English text shall control37
Held: Although the INP traced its roots to the PC, the 60-year Extrinsic aids
old mandatory age of retirement is only applicable to the
local police component of the INP and not to the PC. When 1. Intent or spirit of law
the whole law is examined, the INP and PC are not the same
2. Policy of law similar
entity. Several provisions of the law distinguished PC from
INP, hence, taken into its context, there is no need for the 3. Purpose of law
assailed provision to have a categorical restriction since 4. Dictionaries
other provisions have differentiated the two. The PC before 5. Consequences of various constructions
the enactment of said law were already retirable at 56, while 6. Presumptions
the local police at 60; and these were governed by different
7. Legislative history
laws.
8. Contemporaneous construction
Punctuation mark
The semi-colon (;) indicates separation in the Intent or spirit of the law
relation of the thought. But what follows it - Considered as the law itself, hence, it is the
should be related to the one preceding it. controlling factor, the leading star and guiding
Although both used for the same purpose, the light in the interpretation of a statute
semi-colon is more pronounced and is greater - “For what is within the spirit is within the
in degree than that of a comma (,). Neither is statute although it is not within the letter
used to introduce a new idea. thereof...”
- A period (.) is a mark used to indicate the end
of the sentence. Fabella v. CA
Several teachers were charged with administrative cases for
- Aids of low degree and can never control
joining mass actions demanding payment of bonuses and
against the intelligible meaning of the written
allowances. After an administrative hearing, a committee
words found them guilty and ordered their dismissal.
- Argument based upon these alone is not Held: RA 4670 was not repealed by PD807. Hence, there
persuasive should be a representative of the teachers in the committee
Reason: Punctuation marks are neither a part of hearing administrative cases. Mere membership of teachers
the statute nor the English language. When used: in an organization does not make them authorized
If it gives the statute a meaning which is reasonable representatives. The intent of this provision is the right of
and in accord with the the teachers to be heard and their protection of their rights
will of the legislature when facing administrative charges. The committee,
therefore, has no jurisdiction.

Capitalization of letters
Policy of the law
- Aids of low degree; almost have the same
- A construction which would promote the policy
reasons as that of punctuation marks
of the law should be favoured than that would
defeat it
Headnotes of epigraphs
- In a statute of doubtful meaning, giving a
- Index to the contents of the
construction that will promote public
provisions in a section
policy
- Prefix to a section or chapters
of a statute for ready reference
Purpose of the law
- Not entitled to much weight Reason: Not part
- The purpose of the law or the mischief
of the statute, they are mere catchwords of
intended to be removed or suppressed are

36The case also talked about the principle of problem with this is that the specific word came first.
ejusdem generic, where general words 37 Book I, Chapter 4, Sec. 20 (AC). In the interpretation of a law or

cannot be given their generic meaning if administrative issuance promulgated in all official languages, the English text
followed by specific words: (1) managing shall control, unless otherwise specially provided. In case of ambiguity,
head, (2) directors, (3) partners. The omission or other mistake, the other texts may be consulted.

JUSTIN SUCGANG | BLOCK 2 ATTY. WIGBERTO TANADA, JR.


-
important factors to consider in construction of
statutes
The purpose is more important than the rules
of grammar and logic
- But courts cannot assume some purpose not
expressed

CEMCO Holding v. National Life Insurance CEMCO bought


several shares of two companies. This resulted to its
majority control of UCHC, and in turn, as one of the principal
stockholders, an indirect acquisition of the majority shares
of UCC itself. Respondent, feeling aggrieved because of loss
of market value, charged CEMCO for non-compliance to the
mandatory tender offer rule. Held: The purpose of said rule
is to give minority stockholders a chance to exit the company
under reasonable terms and sell their shares as that of the
majority holders. Even if the acquisition is below the
threshold, if it will result to a majority control, the tender
offer rule should apply.

Dictionaries38
- Legal, scientific, general dictionaries
- Used when statutes do not define the words or
phrases used and the purpose or context in
which the words are employed
- But definitions are not binding to courts
General rule: Dictionaries usually define words
in their natural, plain and ordinary acceptance
and significance Presumption: Lawmakers,
ordinarily untrained philologists and
lexicographers, use words in their common
meaning Exception: When the statute has
defined the words used and/or the legislature
has intended a technical or special legal
meaning to these words.

Consequences of various constructions


- The objective should always arrive at a
reasonable and sensible interpretation in accord
with the legislative intent General rule:
Construction that will lead to injustice or hardship,
result in absurdity, defeat the legislative intent or
spirit, preclude accomplishment of legislative
purpose or object, render certain words or phrases
a surplusage, nullify the statute or make any of

38 See PRC v. De Guzman summary above

JUSTIN SUCGANG | BLOCK 2 ATTY. WIGBERTO TANADA, JR.


its provisions nugatory should be Legislative debates, views and
rejected or avoided. deliberations40
Presumption: The legislature did not intend to - Actual proceedings of the legislative body
cause injustice in enacting statutes. Exception: (committee reports of legislative investigations
When the law is clear and unambiguous. and public hearings, sponsorship speech,
debates and deliberations)
Presumptions - The discussions on the bill may contain the
- Based on logic, experience and common sense. meaning which was put to the provision
- In absence of compelling reasons to rule - The views expressed by the legislators are not
otherwise, doubt should be in favour of the controlling in the interpretation as to the bill’s
presumption on that matter. purpose, meaning or effect Reason: Statements
made by a legislator during floor deliberations
Legislative history do not reflect the views of the entire house. It
- The origin and history of the statute is impossible to determine with authority what
- Refers to all its antecedents from its inception construction was put upon by members of the
until its enactment into law. Congress who passed the bill. Those who did
- Covers the period and steps done from the not speak may not have agreed with those who
time the bill us introduced until it is did; and those who spoke might differ with
finally passed by the legislature each other
(Manila Jockey Club v. Games and
a. President’s message to legislature Amusement Board) Exceptions:
b. Explanatory note 1. There are circumstances indicating a meaning
c. Legislative debates, views and other than that expressed by a legislator
deliberations 2. Views expressed where conflicting
d. Reports of commissions 3. Intent deducible from such views is not clear
e. Prior laws from which statute is based 4. Statute is free from ambiguity
f. Amendment of the statute 5. When the legislator is not a member of the
congress that enacted it
g. Adopted statutes
h. Conditions at the time of
Manila Jockey Club v. Games and Amusement Board
enactment
A law was passed increasing the number of days the PCSO
i. History of the times can conduct races. However, it did not specify when these
days were. The GAB resolved it by reducing the number of
President’s message to legislature39 days private entities can hold their activites.
- The President’s address usually contains Held: The statement of individual members is not binding for
it may only reflect the opinion of that member alone. It may
proposed legislative measures.
not reflect the intent of all those who passed the law.
- It indicates his thinking on the proposed
legislation which, when enacted into law, CASCO Philippine Chemical Company v. Gimenez CASCO filed
follows his line of thinking. for refund for the tax they paid for the separate importation
of urea and formaldehyde following the resolution issued by
Explanatory note the Monetary Board. Held: Although the bill approved in
- Short exposition or explanation accompanying a Congress contained “urea and formaldehyde,” and the
proposed legislation by its author or statements made by some members clearly expressed the
proponent. exemption of separate importation, individual statements do
not reflect the view and intent of Congress as a whole in
o Contains reason or purpose of the bill o
passing said act. The enrolled bill stating “urea and
Arguments advanced by the author in
formaldehyde” is conclusive. If there was a mistake in
urging its passage Exception: The court may printing, a curative amendment must be passed.
not use the explanatory note as
basis for giving a statute a Reports of commissions41
meaning that is inconsistent
- Usually present in the codification of laws for
with what is expressed in the
they compile and collate all laws on a particular
text of the statute. It is resorted only for
subject and prepare a draft of the proposed
clarification in case of doubt.
code

39 Art. VII, Sec. 23 (Const). The President 40 In NAPOLCOM v. De Guzman, the records of the bicameral conference
shall address the Congress at the opening of committee clearly showed the legislative intent to exclude the PC from the
its regular session. He may also appear INP
before it at any other time. 41 In Civil Code and Revised Penal Code; Constitutional Commission

JUSTIN SUCGANG | BLOCK 2 ATTY. WIGBERTO TANADA, JR.


Prior laws from which a statute is Held: The 1997 Revised Rules of Court has
based deleted said provision found in the earlier code. Hence, the
- Part of the antecedent of the statute involved prevailing rule should be obeyed.
are prior laws on the same subject
Laguna Lake Development Authority v. CA
- Especially applicable in the interpretation of
LLDA was charged by law to maintain and protect the
codes and revised or compiled statutes
Laguna de bay. After the passage of the Local Government
- Shows the legislative history that may clarify Code, several mayors who have territorial jurisdiction in
the intent of the law or shed light on its some parts of the lake issued business permits to construct
meaning and scope fish pens in said area. As a result, LLDA ordered the
demolition of the fish pen. Held: It cannot be said that the
Amendments of statutes * Applies when the latter law (LGC) has repealed the LLDA charter, it being the
deleted words or phrases are not surplusage or most recent one. The LLDA is more specific as compared to
when the intention is clear to change the previous the local government code, which generally talks about the
municipal waters.
meaning of the old law.
GSIS v. City Assessor of Iloilo
Change in phraseology by amendment
Private respondent bought two parcels of land named to
- This indicates the intent to change the meaning GSIS in an auction. Said lands were foreclosed for failure to
of the provision from that it had originally pay real property tax. GSIS wanted to annul the judgment of
- Where there is a showing that a statute has entry of title in her name over the lands claiming that it was
undergone several amendments and each exempted from local taxes. Held: It cannot be said that the
amendment used different phraseology, the new GSIS charter has repealed the Local Government Code
deliberate change of words is an indication of provision regarding collection of taxes. Although the titles of
the intent to change the meaning (Portillo v. land are named under GSIS, it has already conveyed and
alienated the lands to private persons. Hence, the lands are
Salvani) Presumption: There must be some
now subject to taxes. In this way, the two laws are
purpose in making changes which should be
reconciled. And even if it does repeal the LGC, it cannot be
ascertained and given effect. applied retroactively especially if a vested right will be
prejudiced.
Amendment by deletion
- Deleting certain words or phrases in a statute Ty v. Trampe
indicates the legislative intent to change its Ty, a registered owner of the land, received a notice of
meaning assessment respecting certain real properties. Held: There is
Presumption: The legislature would not have made no express repeal especially because the statute has
the deletion had the intention been not to effect a provided a clause enumerating the laws it shall repeal.
change in meaning. Before determining if there is an implied repeal, the Court
should try to harmonize said laws. The assessor’s action was
set aside since he should have coordinated with other city
Exception to the rule that amendment indicates
assessors before determining the tax increase.
change: Does not apply where the intent is clear
that the amendment is precisely to plainly express
Adopted statutes
the construction of the act prior to its amendment
because its language is not sufficiently expressive - Foreign statutes adopted in this country form
of such construction. 42 part of its legislative history. General rule: The
Reason: Remember that in codification, revision interpretation and decision of foreign courts are
and compilation of laws, condensation and brevity given great weight if the local statute was
is necessary. Words that do not materially affect patterned after or copied from those of another
the statute are omitted. country.
Presumption: In adopting foreign statutes, the
Tung Chin Hui v Rufus B. Rodriguez Petitioner was arrested legislature is deemed to adopt also the previous
for using a tampered passport which has been cancelled by judicial construction and practical application of
Taiwanese authorities. He filed a said statute in that country.
petition for habeas corpus to an Exceptions:
RTC and was granted. After the denial 1. Where there is material difference between
of a motion for reconsideration, the foreign and local law
respondents filed a notice of appeal of judgment. Petitioner
2. Where the foreign construction is patently
filed an opposition since the appeal was filed long after the
48-hour period prescribed for habeas corpus cases by the
erroneous or has not been settled
pre-1997 Rules of Court.

42In Enrique v. CA, the MSPB law did not to reconcile the conflicting provisions. Only if the repugnancy is
render the CSC law invalid since the two can irreconcilable that we can say that there has been an implied repeal.
be harmonized. The first duty of the court is

JUSTIN SUCGANG | BLOCK 2 ATTY. WIGBERTO TANADA, JR.


3. Where foreign construction is not v. Rafferty), weight given increases as the
reasonable, not in harmony with justice, public period in which it is followed and observed
policy and other local statutes lengthens and its acceptability widens.
4. Where the local court has given its own - So long as it continues and Congress has not
construction to said statute seen fit to repeal or change the law

Conditions at the time of enactment Presumption: Executive officials are presumed to have
- Physical conditions of the country and the familiarized themselves with all considerations
then circumstances that may affect the pertinent to the meaning and purpose of the law, and
legislative intent to have formed an independent, conscientious and
- The Court should place itself in the situation competent expert opinion (Asturias Sugar Central, Inc.
of the legislature during that time v.
Presumption: Statutes do not operate in a vacuum. Commissioner of Customs)
In enacting a statute, the legislature is presumed to
have taken into account the Reasons:
existing conditions of things at the time of 1. Respect due the government agency or officials
enactment charged with the implementation of the law

History of the time


- Almost similar to taking into consideration the
condition at the time of enactment 2. Competence, expertness, experience, and
- The law, being a manifestation of social culture informed judgment, and the fact that they draft
and progress, must be interpreted taking into the law they interpret
consideration the stage of such culture and 3. Need for certainty and predictability in the law
progress including all concomitant
circumstances Presumption: Law is not a Exceptions:
watertight compartment sealed or shut off 1. No ambiguity in the law (Regalado v Yulo)
from the contact with the drama of life which
2. Construction, clearly erroneous is null and void
unfolds before our eyes. It is not a cloistered
(Molina v. Rafferty)
realm but a busy state in which events are held
3. The court has previously given the statute a
up
different interpretation (Asturias Sugar Central,
to our vision and touch our elbows.43
Inc. v. Commissioner of Customs)
Contemporaneous construction44
- Also called practical constructions Enrique v. CA
- Constructions placed upon the statutes at the Petitioners were charged for helping examinees in the civil
time of, or after, their enactment by the service exam in consideration of a payment. The CSC
executive, legislative or judicial authorities rendered a decision dismissing them.
Maxim: contemporanea exposition est optima et Held: Although there is a law creating a Merit System
fortissimo in lege (the contemporary Protection Board under the CSC vesting it with jurisdiction to
construction is strongest in law) hear and decide administrative cases, CSC also has a
concurrent original jurisdiction. The MSPB law did not
amend the CSC law. Great weight is given to the CSC
*What is commonly understood as contemporaneous Memorandum Circular stressing the concurrent jurisdiction
construction is that one placed upon the statute by an of the two bodies.
executive or administrative officer.
*With the duty to enforce the laws come the *But erroneous construction does not prevent
interpretation of its ambiguous provisions. correction:
- The doctrine of estoppels does not preclude
Weight given to correction of the erroneous construction by the
contemporaneous construction officer himself, his
- It is entitled to great weight successor, or the Court.45
and respect by the courts in the
- A person who has relied upon such
interpretation of ambiguous provisions of law.
construction may not prevent the application
- Especially if the construction is followed for a of the new interpretation.46
considerable period of time (Molina

43Wortham v. Walker Tex tasked to implementation of the Securities Act.


44 45 PLDT v. Collector of Internal Revenue
In CEMCO Holdings, Inc. v. National Life
Insurance, the contemporaneous
construction of the SEC to the mandatory 46 Koppel (Phils.) v. Yatco
tender offer rule was sustained since it was

JUSTIN SUCGANG | BLOCK 2 ATTY. WIGBERTO TANADA, JR.


- This who have benefited may not *Legislative interpretation
prevent correction; or excuse themselves from 1. Prescribes rules of construction or indicating
complying with the corrected one; or set up as how its provisions should be construed in an
a legal obstacle against recovery from them of interpretative or declaratory clause of a
what they received on the basis of an statute
erroneous construction47 2. The legislature may also define the terms used
General rule: Erroneous construction creates no vested in the statute
rights on the part of those who relied upon them, and 3. It may also pass a resolution indicating its sense
followed such construction. Reason: A vested right or intention in a statute
may not arise from a wrong interpretation of a law by 4. May also take the form of legislative approval:
an administrative or executive officer whose primary a. Re-enactment of a statute
duty is to enforce, and not to construe the law.48
previously given a contemporaneous
Exception: This is not absolute and admits
construction is a persuasive indication
exceptions in the interest of justice and fair play.49
of legislative approval50

ABS-CBN v. CTA
b. Using words similar in import to the
A memorandum circular interpreting the National language of an earlier law which has
Internal Revenue Code was release. Following this, ABS-CBN received a practical interpretation
dutifully paid the taxes according to the formula set. A new c. Amending a prior statute
law was passed amending the assailed provision. A new without providing anything which
order was released by the commissioner of internal revenue would restrict, change or nullify the
revoking the earlier circular for being erroneous for lack of previous contemporaneous
legal basis. Because of this, they demanded petitioner to pay construction placed upon it
the deficiencies based on the new formula. Held: Although
d. Appropriating money for the
the government is not estopped to collect taxes because of
mistakes of its collectors, it admits of certain exceptions in officer designated to perform a task
the interest of justice and fair play. The order cannot be pursuant to an interpretation of a
retroactively applied if it will prejudice the taxpayer, statute
especially if the latter has followed it in good faith. e. With notice of a previous construction,
the legislature did not do repudiate it,
Kinds of executive interpretations there is implied acquiescence to, or
1. Interpretation by usage or practice- approval of, an executive or judicial
construction made by an executive officer construction of a statute Exception:
directly called to implement the law (circular, The legislature, in indicating its
directive, regulation, opinions and rulings) construction of a law, cannot limit or
2. Advisory opinions- one made by the restrict the power of the Court.
Secretary of Justice in his capacity as chief legal - It may not make a definition conclusive to a
adviser of the government upon request of statute which defined the term or to other
administrative officials who enforce the law. statutes (City of Manila v. Manila
3. Ruling of quasi-judicial agencies- Remnants Co, Inc)
interpretation handed down in an - It cannot validate a law which violates the
adversary proceeding constitution to prevent an attack of it in
the courts (Endencia v. David) When used:
*The common usage and practice under a statute is of While legislative interpretation is not
great value to its construction. controlling, the Court may resort it to
Maxim: optimus interpres rerum usum (the best clarify ambiguity in the language. Such
interpreter of a law is usage) interpretation is entitled to respect,
- Rules and regulations issued by executive or
administrative officer pursuant to law have the
force and effect of laws.
- The administrative agency has
the power to interpret its own
rules; this will become part of the rules

47 Legaspi v. Mathay 50To be applicable, earlier law must be re-enacted and not merely amended
and the contemporaneous construction must be in the form of regulation to
48 Hilado v. Collector of Internal Revenue implement the law and duly published and not merely administrative ruling
embodied in a letter to a specified individual and not published (Alexander
49 ABS-CBN v. CTA Howden & Co., Ltd. v. Collector of Internal Revenue).

JUSTIN SUCGANG | BLOCK 2 ATTY. WIGBERTO TANADA, JR.


especially if the executive Ting v. Velez-Ting
department has similarly construed the Respondent filed for a declaration of nullity of marriage
state.51 52 under Art. 36 of the Family Code. She claimed that her
husband does not only give support to the family but
also is a compulsive gambler, drunkard, and violent. The
Stare decisis
trial court and CA declared their marriage null and void
- The decisions of the Supreme Court applying or after taking cognizance of the psychiatric finding
interpreting a statute is controlling with presented by respondent.
respect to the interpretation of that statute Held: The Court did not overrule the Santos and Molina
and is of greater weight than that of an doctrines. It merely liberalized it since instead of serving
executive or administrative officer as guidelines, these became a straightjacket fitting all
- Desirability of having stability in the law cases into it. The psychiatric reports shall not be
conclusive to the courts and are not condition sine qua
Reason: Interpretation given by the Supreme Court
non for the nullity of marriage. The totality of evidence
forms part of that statute itself and of the legal should be the basis.
system and comes from that branch entrusted with
the duty to construe or interpret the law.53
When not used:
Maxim: stare decisis et non quite movere (one 1. Ruling is an obiter dictum54
should follow past precedents and should not
2. Facts are dissimilar
deserve what has been settled) Exception: It does
not mean blind adherence to precedents. The 3. There is a conflict between the
doctrine, no matter how sound, must be precedent and the law55
abandoned if found to be contrary to law. For it is
more important for the court to be right. *Only Supreme Court can change or
abandon a precedent enunciated by it.
When applicable:
1. Direct ruling- ruling must be categorically *Until it abandons or overrules a
stated on an issue expressly raised by the doctrine, the lower courts are duty bound
parties to obey it. If lower courts feel that the
2. Facts are substantially the same doctrine is against their way of
reasoning, they may state their personal
Lazatin v. Desierto opinion but must decide the case in
Congressman Lazatin was charged by the Fact Finding accordance with
Bureau of the Office of the Ombudsman with
malversation of public funds. The Office of the Special the doctrine and not with their personal 56
Prosecutor (OSP) submitted its resolution to dismiss the view.
charge for lack of evidence. The Ombudsman ordered Tan Chong v. Secretary of Labor
the Office of Legal Affairs to review the OSP resolution.
Two cases are involved having almost similar facts.
The latter recommended proceeding with the case.
Lazatin claimed that, under the Constitution, it is not Both petitioners had a Chinese father and a
within the power of the Ombudsman to prosecute. The Filipino mother and were born in the Philippines:
power is expressly given to the OSP. the first one in San Pablo, Laguna, the other in
Held: Although not expressly enumerated, the Jolo, Sulu. At first, they were declared Filipino
Ombudsman is given other powers necessary to further
citizens by the Court but the Solicitor General filed
its duties. As early as Acop v. Office of the Ombudsman,
the Court already ruled that RA 6770 is not a motion for reconsideration.
unconstitutional, although it subsumed the OSP under Held: The Court has cited a long list of cases
the Ombudsman. Several cases followed the precedent; pertaining to citizenship, in which its decisions
hence, unless there is no clear and compelling reason to
vacillated. Stare decisis cannot apply when the
abandon the doctrine, it should be followed.
precedent and the existing law are in conflict. Since

51For an orderly and harmonious necessary to the decision of the case before it. It is a
interpretation and advancement of the law, remark made, or opinion expressed, by a judge in his decision upon a
the court should, when possible keep step cause “by the way,” that is, incidentally, and not directly upon
with other departments (Yra v. Albana). the question before him, or upon a point not necessarily involved in
the determination of the cause, or introduced by way of illustration,
52In case of an agreement between two or analogy or argument (Delta Motors Corp. v. Secretary of
departments- the executive and legislative-
Labor).
to the meaning of the law, and it devolves
upon the judiciary to give it a deferential
55In Tung Chin Hui v Rufus B. Rodriguez, the Court held that the doctrine
treatment (Bengzon v. Secretary of Justice).
of stare decisis cannot be invoked since there is a new law that took
53
Art. 8 (CC) effect, hence, the rulings of the previous cases shall no longer apply.
54Opinion express by a court upon
56 Canon 18, Canons of Judicial Ethics
some question of law which is not

JUSTIN SUCGANG | BLOCK 2 ATTY. WIGBERTO TANADA, JR.


the jus soli common law principle of the petitioners were not Filipino citizens on the
US did not extend to out jurisdiction and, only ground that their father was of Chinese parentage.
Spanish subjects residing from April 11, 1899 are
considered citizens under the then prevailing
Philippine Bill of 1902, the Court ruled that

ADHERENCE TO, OR DEPARTURE FROM, 5. The law should be applied regardless whether
LANGUAGE OF STATUTE it is unwise, hard or harsh.
6. If the law is clear and free from doubt, it is the
Adherence to the language sworn duty of the court to apply it without fear
or favour, to follow its mandate, and not to
tamper with it.59
Literal meaning, Plain-meaning rule, Verba legis
7. What it decrees must be followed; what it
General rule: The intent of the legislature is found in commands must be obeyed.60
the language of the statute. 8. Where the law is clear, appeals to justice and
equity as to justification to construe
Maxims: it differently are unavailing. 61
- index animi sermo est (speech is the index of
intention) Globe Mackay v. NLRC
- verba legis non est recedendum (from the Private respondent Salazar was found to be close to Saldivar
words of the statute there should be no and, in fact, living with him in an apartment. The latter was
investigated for defrauding petitioner and use of an air-con
departure)
without authorization. Because of this, she was suspended
- dura lex sed lex (the law may be harsh, but it is and, later on, dismissed. Held: The suspension is legal.
still the law) However, there was no sufficient evidence for her dismissal.
- hoc quidem perquam durum est, sed ital ex Hence, following the provision of the law, she was unjustly
scripta est (it is exceedingly hard but so removed and is entitled to reinstatement and payment of
the law is written) Reasons: back wages. The principle of “strained relations” cannot be
- A statute, being the will of the legislature, accepted since her job as systems analyst is not one
characterized as a position of trust. Loss of confidence
should be applied in exactly the way the
cannot also be accepted since it must rest upon a valid basis.
legislature has expressed itself clearly in the
The report made by the company’s internal auditor is one-
law. sided because Saldivar has already resigned, depriving him
- The duty of the court is limited to inquiring into opportunity to defend himself.
the legislative intent and, once this is
determined, to making said intent effective. Cecileville Realty & Service Corp. v. CA Petitioner owned a
parcel of land while private respondent Pascual occupied a
Phrases: portion of it. Despite several demands, Pascual did not
1. What is not clearly provided in the law cannot vacate the land. Held: The defense that Pascual was just
be extended to those matters outside its helping his mother, a tenant, in cultivating the lands cannot
be accepted. The law is clear, only the tenant, and not his
scope.57
relatives, can demand for a home lot suitable for dwelling.
2. Where the statute is clear, plain, and free from
ambiguity, it must be given its literal meaning, RCBC v. IAC
applied without attempted interpretation and BF Homes filed a petition for rehabilitation and suspension
must be taken to mean exactly as what it says. of payments with the SEC. RCBC, as a creditor, requested the
3. Where the law is clear and free from doubt and sheriff to extra-judicially foreclose its real estate mortgage.
ambiguity, there is no room for construction or Despite the ongoing case in the SEC, RCBC filed an action for
interpretation. mandamus to
compel the sheriff to execute a certificate of sale in its name.
4. Courts must administer the law, not as they
Later on, SEC appointed a management committee to assist
think it ought to be but as they
BF Homes with the rehabilitation/liquidation.
find it and without regard to Held: Even if it would seem logical that suspension should
consequences. 58 succeed the application for rehabilitation, the law is clear.

57 Baranda v. Gustillo
60 PNB v. Bitulok Sawmill, Inc.
58 For whether a statute is wise or expedient
61For equity is available only in the absence of law and not its replacement.
is not for the courts to determine (Director
of Lands v. Abaya). Equity is described as justice outside legality, which simply means that it
cannot supplant although it may supplement the law. Aequitas nunquam
59
Go v. Anti-Chinese League of the contravenit legis. Equity never acts in contravention of justice (Aguila v. CFI
Philippines Batangas).

JUSTIN SUCGANG | BLOCK 2 ATTY. WIGBERTO TANADA, JR.


Only when a management committee is Maxims:
appointed will other actions and claims shall be suspended. - ratio legis et anima legis (the reason of the law
is its soul);
Segovia v. Sandiganbayan - ratione cessat lex et cessat lex (when the
Segovia, and other petitioners who were suspended, were
reason of the law ceases, the law ceases)
designated to the Contracts Committee for a project of
NAPOCOR in Mindanao. They had disqualified the two - verba intentioni, non e contra, debent inservire
lowest bidders. Because of this, Urban, the second lowest (words ought to be more subservient to the
bidder complained and charged them for violations of the intent and not the
Anti-Graft and Corrupt Practices Act. As a result, they were intent to the words)
suspended during the pendency of investigation. Held: The
law clearly provided that all officials, during the pendency of People v. Almuete
a criminal prosecution, shall be meted with a preventive Defendants were accused of pre-threshing a portion of the
suspension. It is mandatory and not discretionary on the part harvest without notice to and consent of the landowner.
of Sandiganbayan; this is to prevent them from influencing They were charged for violating the Agricultural Tenancy Act.
the process either by intimidating the witnesses or Held: The case was dismissed since the law from which the
tampering with evidences. crime was founded has been effectively repealed by the
Agricultural Land Reform Code. A similar case was also
People v. Mapa dismissed by the SC since the law was already in force at the
Mapa claimed that he was a secret agent of Governor Leviste time of the commission of the crime.
when he was charged for illegal possession of unlicensed
firearms. When used:
Held: Although it was earlier held that secret agents may be
- A statute must be capable of construction or
exempted from possession of firearms since they do the
interpretation. The court must use every
work of peace officers, the Revised Administrative Code
clearly enumerated those who may be exempted, and secret authorized means to ascertain the intent of the
agents were not included. statute and give it an intelligible meaning. The
court may even depart from the language of a
Gan v. Reyes statute if to do so will enable to effectuate
Reyes filed an action for support, claiming that she and Gan legislative intent and purpose.
had a lovechild. The trial court ruled in favour of Reyes and - If a language is capable of more than one
ordered the immediate execution of the support. Gan meaning is to be taken in such sense as to
challenged the ruling and still withheld support. harmonize with the intention and object and
Held: Child support is immediately executory and cannot be effectuate the purpose of the enactment.
stayed by an appeal unless provided by the trial court. This
has been clearly stated in the Rules of Civil Procedure.
When not used: Where a statute totally fails to express
a meaning, a becoming sense of judicial modesty
Largado v. Masaganda
forbids the court from assuming and, consequently,
Private respondent Delos Reyes filed a petition to the Justice
of the Peace asking for the guardianship of petitioner’s
from supplying a meaning to it.
children. It was decided in his favour but upon appeal to the Hence, the statute is necessarily inoperative.
CFI, it was dismissed since the Judiciary Act of 1948 said the
justices of the peace may not extend to the appointment of Limitation: Applies only when there is ambiguity in the
guardianship. Held: The decision of CFI is affirmed. The law language employed in the law. Where the law is clear
was clear. Although it was contended that the term and free from ambiguity, the letter of the law is not
“guardian” was only due to an oversight, as opined by the disregarded on the pretext of
Justice Secretary, the mistake cannot be corrected by pursuing its spirit.62
executive fiat but by legislation, which Congress has done in
enacting RA3090 correcting said statute. But this cannot be
given retroactive effect. Phrases:
1. The thing which is within the intention of the
Departure from literal interpretation statute is as much within the statute as if it is
were within the letter, and a thing which is
within the letter of the statute is not within the
Ratio legis
statute unless it be within the intention of the
- Interpretation according to the
lawmaker.63
spirit of reason of the law
2. Intent is the spirit which gives life to a
General rule: The spirit or intention of a statute prevails legislative enactment.64
over the letter of it.
How done:

62 De Jesus v. City of Manila


64 Torres v. Limjap
63 US v. Go Chico
JUSTIN SUCGANG | BLOCK 2 ATTY. WIGBERTO TANADA, JR.
In correcting a clerical error or not the feminine, includes all genders,
obvious mistake, the court is not indulging in a judicial unless otherwise provided or implied in the
legislation, it is merely endeavouring to rectify and context.
correct a clearly clerical error in the wording of the law
in order to give due course and carry out the evident Rufino Lopez and Sons, Inc. v. CTA Petitioner imported a
intent.65 hexagonal wire netting from
Germany and the Collector of Customs assessed it. Failing to
secure a reassessment, petitioner appealed to the CTA. The
- Supplying legislative omission
case was dismissed for lack of jurisdiction.
Where literal import of the language of a Held: There was a clerical error in the statute. Following the
statute shows that words have been omitted literal interpretation, the statute has lodged with the CTA
that should have been in the statute to carry the appellate jurisdiction to decisions of the Commissioner of
out its intent and spirit, clearly ascertainable Customs, but no person affected by the decision of him may
from the context, the court may supply the appeal to the CTA, only those made by the Collector of
omission to make the statute conform to the Customs. Collector in Section 11 should be read as
obvious intent of the legislature or to prevent Commissioner; this is not judicial legislation but only carrying
the act from being absurd.66 out the intent of the legislature.

- Correcting clerical errors


Construction according to the spirit:
The court, in order to carry out the obvious
A statute may therefore be extended to cases not
intent of the legislature, may correct clerical
within the literal meaning of its terms, so long
errors, mistakes or misprints which, if
as they come within its spirit or intent.71
uncorrected, may render the statute
meaningless, empty or nonsensical or would
defeat or impair the intended operation.67 1. Construction to accomplish purpose Statutes
should be construed in the light of the object to be
- Surplusage and superfluity disregarded
achieved and the evil or mischief to be
Where a word, phrase or clause in a statute suppressed.72 Construction which gives effect to
is devoid of meaning in relation to the context the evident purpose sough to be attained by the
or intent of the statute, or where it suggests a enactment of the statute must be followed.73
meaning that nullifies the statute or renders it Hence, as between two statutory interpretations,
without sense, these may be rejected as that which better serves the purpose of the law
surplusage and entirely ignored.68 should prevail.74
- Redundant words may be rejected Reason: Laws are not just mere composition, but
Although it is a general rule to give every have ends to be achieved and that the general
part of the statute a meaning, it is not the purpose is a more important aid to the meaning of
obligation of the court to give a redundant a law than any rule which grammar may lay down.
word or phrase a special significance. A use of When not used: cessante ratione legis, cessar et
word or phrase, treated as a mere reiteration ipsa lex (when the reason of the law
or repetition of other language in the statute, ceases, the law itself ceases)
may be rejected.
- Obscure or missing word or false description 2. Construction to avoid absurdity Statutes may
may be disregarded Lingual imperfection in the be extended to cover cases not within the literal
drafting of the statute should never be meaning of the terms, if their exact and literal
permitted to hamstring judicial search for import would lead to absurd or mischievous
legislative intent. The court may disregard result75 and nonsensical results. 76
loose or obscure words in order to arrive at the Presumption: Congress could not have intended
real meaning.69 absurd interpretation of the law. Maxim:
- Number and gender of words interpretatio talis in ambiguis simper fienda est ut
A word in plural may include the singular, and evitetur inconveniens et absurdum (where there is
vice versa.70 The masculine, but

65 Lamp v. Phipps 71 Rufino Lopez and Sons, Inc. v. CTA

66 72 LVN Pictures, Inc. v. Philippine Musicians Guild


Barret v. Union Bridge, Co.

67 73 Sanciangco v. Rono
Griffin v. Greene

68 74 Salenillas v. CA
Demafiles v. COMELEC

69 75 Rufino Lopez and Sons, Inc. v. CTA


Salaysay v. Castro

70 76 People v. Duque
Santillon v. Miranda

JUSTIN SUCGANG | BLOCK 2 ATTY. WIGBERTO TANADA, JR.


ambiguity, such interpretation as will Casela, in fact, he secured 3 writs of
avoid inconvenience and absurdity is adopted) execution. Casela only wanted to delay and prolong the
execution.
Paras v. COMELEC
Petitioner was then the incumbent barangay captain. A year Cosico v. NLRC
after his election, almost more than 25% of his constituent The Assistant Station Manager position of Eva Air operations
filed a petition for recall. Petitioner contended that the in Manila, which Cosico was holding, was abolished because
move for a recall is barred since by next year, an SK election of its cost-cutting measure. The operation target of 60
will be held, which he interprets as a regular local election passengers per flight was not reached, it was found out that
(the LGC prohibits any more for recall a year after and only 25 average passengers were there for a flight. Because
before a local election). of this, he sued Eva Air for illegal dismissal in the labor
Held: To accept the literal interpretation of petitioner leads arbiter and ruled in his favour. Eva Air elevated the case in
to absurdity. This will expressly prohibit any recall move and the NLRC and posted a surety bond, but only amounting to
defeat the intent of Congress. The local election should be the sum of back wages and 13th month pay. Held: The Court
referred to as the one from which his position will be held that to require Eva Air to post a bond inclusive of
opened- in this case, barangay captain. exemplary and moral damages and attorney’s fee is unjust,
excessive and has the effect of depriving their right to
Chartered Bank of India v. Imperial appeal.
The Philippine National Bank filed a petition in the CFI
Manila to compel Umberto de Poli et al. to deliver to them Salvacion v. Central Bank
the goods and merchandise described in the mortgage. As a Petitioner was raped several times by private respondent
result, the sheriff has seized the goods described. Within 24 Bartelli, a tourist. On the day of his hearing for petition for
hours after the sheriff has taken possession of said goods, bail, Bartelli escaped from jail. Hence, a notice of
petitioners filed a petition to declare Umberto de Poli garnishment was delivered to Chinabank. However,
insolvent and was granted. The court ordered the sheriff to Chinabank contended that the account cannot be garnished
take possession of the goods and merchandise which had since RA 1405 prohibits the garnishment of a foreign
already been attached by him in the action instituted by the account. Held: Although the law is clear, it cannot be
PNB against de Poli. invoked since it will lead to injustice. The intention of the law
Held: Although creditors should be treated equally in the is for economic reasons, to prevent withdrawal of dollar
distribution of goods of an insolvent, PNB is a preferred reserves since foreign investments were minimal. However,
creditor for having a secured holding such as mortgage, lien in Bartelli’s case, he was just a transient and will maintain
or pledge. To construe a provision without reference to its the account for a short period of time. To uphold the law is
first part, would result to an absurd and unreasonable to deny justice to a victim of a foreigner who will just stay
situation where his rights are preserved but he cannot temporarily by not availing of damages.
participate in the election of an assignee in an insolvency
proceeding, or surrender a property delivered to him as 4. Construction in favour of right and justice
security of said lien for the benefit of all creditors of the Any doubt in the construction of a statute should
insolvent. be resolved in favour of right and justice.
Maxim: jure nature aequum est neminem cum
3. Construction to avoid injustice The alterius detriment et injuria fiery locupletiorum
interpretation to be adopted is that which is free (when the statutes are silent or obscure, the
from evil or injustice. Presumption: The solution is that would response
legislature, in enacting a law, did not intend to to the vehement urge of conscience)
work a hardship or an oppressive result.
Maxim: interpretation quae vitio caret (that Alonzo v. IAC
interpretation is to be adopted which is free Private respondents were siblings who inherited a piece of
from evil or injustice) land. Two of them sold their part to spouses Alonzo. Later,
the son of the spouses built a house after getting married.
Casela v. CA One of the siblings brought the action to recover said
Petitioner’s house was to be demolished after Magsaysay portion for not being informed of the sale (since the land is
secured a writ of execution from the CAR. But he did not under a co-ownership). Held: Although the law requires a
vacate the land and even instituted written notice of sale from the vendor, equity and justice
a petition for payment to his house should also be considered. For several years, petitioners
and other improvements in the CFI. have been living in said area. It is improbable that private
After two more writ of executions, respondents would not notice that there are people not one
CAR ruled that its earlier decision cannot anymore be of them living in their inherited land. The son of the
executed since the 5-year reglementary period has already petitioners has built his house receiving no complaint from
lapsed. the siblings. Hence, the prescription of the right of
Held: An interpretation that will lead to injustice should be redemption has started sometime during the 1960s (time
avoided. Although 5 years have passed since the decision when lots were sold).
has become final, the time when the writ cannot be served
should be deducted. This is because Magsaysay has not been
sleeping on his rights and exhausted all legal means to eject

JUSTIN SUCGANG | BLOCK 2 ATTY. WIGBERTO TANADA, JR.


5. Construction to avoid danger to who have rendered years of service to the
public interest government should be deemed included.
Where great inconvenience will result, or great
public interest will be endangered or sacrificed, or Limitations:
great mischief done, from a particular construction 1. Since it excludes what is merely plausible,
of a statute, such construction is to be avoided. beneficial or desirable, this may not be used to
justify the inclusion in a statute of what to the
6. Exemption from rigid application of law court to be wise and just, unless it is at the
Equity as well as exceptional situations in a case same time necessarily and logically within its
may require a departure from established rules. terms.
Where the rigorous application of the law would 2. May not be used to support an interpretation
result in preventing a fair and impartial injury into destructive of the object or purpose of the law.
the actual facts of a case, the exigencies of justice 3. May not be contrary to the Constitution or to
demand that the general rule should yield to existing laws.
occasional exceptions wherever there are weighty
reason for it. Necessary implications:
Maxim: summum jus, summa injuria (the rigor of 1. Remedy implied from a right
the law would become the highest The existence of a right in favour of a
injustice) person implies a corresponding obligation on
the part of another who violates such right,
7. Law does not require the impossible A statute and entitles the former to a remedy to assure
may not be construed as to require compliance its observance and vindication.79 Maxim: ubi
with what it prescribes cannot, at the time, be jus, ibi remedium (where there is right, there is
legally accomplished Maxim: nemo tenetur ad a remedy for its violaton)
impossible (the law obliges no one to perform an 2. Grant of jurisdiction
impossibility) Impossibilium nulla obligation est Jurisdiction to hear and decide cases is
(there is no obligation to do an impossible thing) granted by the Constitution or a statute. But it
cannot be implied from the language of the
8. Implications statute, in the absence of a clear legislative
Doctrine of necessary implications- what is implied intent. But the grant of it carries all necessary
in the statute is as much a part of it as that which is and incidental power to employ all writs,
expressed.77 Every statute is understood, by processes and other means essential to make
implication, to contain all such provisions as may the jurisdiction effective.80 Hence, where a
be necessary to effectuate its object and purpose.78 court has jurisdiction over the main cause of
Maxims: ex necessitate legis (from the necessity of action, it can grant reliefs incidental to it, even
law) in ei quod plus sit, semper inest et if these are outside of its jurisdiction.
minus (the greater includes the lesser) 3. Grant of power
Where a general power is conferred or
Chua v. CSC duty enjoined, every particular power
Petitioner Chua, a coterminous to a project in the necessary for the exercise of one or the
National Irrigation Administration (NIA) for a span of 15 performance of the other is also conferred.
years, wanted to avail the RA6683 providing for early These incidental powers are those which are
benefits and separation from government services.
necessarily included in, and are therefore of
Respondent CSC did not grant her petition stating that she is
lesser degree than the power granted.
not a career and non-career employee, her position is not
included in its regular plantilla. - The power to establish an office
Held: To extend the benefits to the non-career employees includes the power to abolish it81
such as casual and emergency employees and exclude - The power to approve a license
coterminous ones will violate the equal protection clause of includes the power to revoke it82
the constitution since they are similarly
- The power to deport includes the
situated. Hence, in order to give life
to it, the doctrine of necessary
power to order the arrest of
implications should be applied, and coterminous employees, undesirable aliens83

77 In re Dick 81 Castillo v. Pajo

78 Go Chico v. Martinez 82 Grodon v. Veridiano

79 Batungbakal v. National Development Co. 83 Qua Chee Gan v. Deportation Board


80 Suanes v. Chief Accountant of the Senate

JUSTIN SUCGANG | BLOCK 2 ATTY. WIGBERTO TANADA, JR.


- The power to regulate construction. Which meaning should be
telephone service includes the power given a word or phrase rests upon what the legislature
to establish and operate by themselves intended.
a telephone service84
- The power conferred to an 1. Ordinary meaning
administrative officer to issue rules and 2. Statutory definition
regulations includes the authority to 3. General words
delegate to subordinate officer the a. Where the law does not distinguish
performance of a particular 4. Commercial or trade meaning
function85 5. Technical or legal meaning
4. What cannot be done directly cannot be 6. Identical terms
done indirectly86 Maxim: quando aliquid 7. Purpose of statute
prohibetur ex directo, prohibetur et per 8. Relation to other provisions
oliquum ratio legis
5. There should be no penalty for compliance 9. Dictated by context
with law
a. Disjunctive and conjunctive words
If an act is faithfully done in compliance
with the law, simple logic, fairness and reason
Ordinary meaning
countenance cannot
allow an imposition of penalty - The general rule is that words should be given
their plain, ordinary, and common usage
meaning89
Limitations to implications:
1. Grant of power excludes greater power - Natural, ordinary, commonly accepted and most
obvious signification Presumption: Lawmakers
2. What is implied should not be against the
law employed words in their ordinary and common
use and acceptation. Since the Court should not
3. Authority to charge against public funds
presume that the lawmaking body does not know
may not be implied
the meaning of the words and the rules of
4. Doctrine of pari delicto Maxims:
grammar.
ex dolo malo non oritur action (no
man can be allowed to found a claim upon his Statutory definition
own wrongdoing or inequity) nullus
- Statutes sometimes define particular words and
commodum capere potest de
phrases
injuria sua propria (no man should be allowed
- In this case, the legislative definition controls the
to take advantage of his own wrong)
meaning of the word, irrespective of the ordinary
in pari delicto potior est condition
or common meaning90
defendentis
Exceptions: - But it is not a usurpation of the court’s function to
interpret the laws, since it merely legislates what
1. If the violation is done against an avowed
should form part of the law
fundamental policy or public interest87
itself91
2. When the transaction is not illegal but
Limitations:
merely prohibited and the prohibition is
1. Controlling in the act concerned and not in
designed for the protection of one party88
other statutes even if the same word or term is
INTERPRETATION OF WORDS AND PHRASES used
2. If the application creates obvious incongruities
How to define words and phrases in the language of the statute, destroys the
purpose, or
Words or phrases may have becomes illogical
an ordinary, generic, restricted,
technical, legal, commercial or General words
trade meaning, which may be - Words of general significance in a statute is to be
defined by the statute itself or have received a judicial taken in its ordinary and comprehensive sense

84 PLDT v. City of Davao


88 Barsobia v. Cuenco
85 89 Mustang Lumber, Inc. v. CA
Esperanza v. Castillo

86 90 People v. Buenviaje
Ligot v. Mathay
91 De Ponce v. Sagario
87 Enrique Yuchengco, Inc. v. Velayo
JUSTIN SUCGANG | BLOCK 2 ATTY. WIGBERTO TANADA, JR.
- A general word should not be given a does not distinguish, courts should not
restricted meaning where no restriction is distinguish) Presumption: The legislature made
indicated no classification in the use of a general word or
- If a word in a statute has both restricted and expression
general meaning, the general must prevail over Exception: Where there are facts or circumstances
the restricted unless the nature of the subject showing that the legislature
matter or the context in which it is employed intended a distinction or qualification
clearly indicates that the limited sense is intended
Exception: If the word is given a different or restricted Ramirez v. CA
meaning Private respondent allegedly insulted and humiliated
Maxims: generalia verba sunt generaliter petitioner in a manner offensive to her dignity and
intelligenda (what is generally spoken shall be personality. Because of this, she instituted a civil case for
generally understood) generale dictum generaliter damages where she presented a verbatim transcript of the
event because she recorded said conversation. With this,
est interpretandum (a general statement is
private respondent filed a criminal case against Ramirez for
understood in a general sense) violating the antiwiretapping act or RA 4200. Petitioner
contended that she is exempted since she was a party to the
People v. Santiago conversation.
Santiago was accused of libel for besmirching the name of Held: The law clearly provided that “any person” not
then Mayor Arsenio Lacson with malicious intent. He authorized by all parties to any private communication is
contended that it was only oral defamation, in which, action liable. It did not make any distinction whether or not a
has already prescribed. Held: Amplifier, used by defendant, person to be penalized should or should not be a party to
is not similar to radio. The latter involves transmission of the conversation. Hence, even a person, party to a
electromagnetic waves without wires, while the former uses conversation, records it without the others’ knowledge is
a conducting wire intervening between the transmitter and considered a violator.
the receiver. Radio cannot be then interpreted to include
amplifier. And using the associated words, it should be Commercial or trade meaning
construed as to be characterized with permanence.
- Words and phrases, which are common use among
merchants and traders, acquire trade or
*Progressive interpretation
commercial meanings which are generally accepted
- Generic term should include not only peculiar in the community in which they have common use
conditions obtaining at the time of its
- Hence, the understanding of the trader determines
enactment but those that may normally arise
the construction to be given
after92
- Usually applicable to tariff laws and laws of
- Apply to all cases that come within the terms commerce
and its general purpose and policy, existing Exception: Legislative intent to the contrary Examples:
subsequent to its passage
1. “Dispose of” is to sell95
- Keeps legislation from becoming ephemeral
2. “Gross value in money” is gross selling price96
and transitory

Where the law does not distinguish Technical or legal meaning


- Corollary of the principle that general words - Words used in technical sense, or have been
and phrases in a statute should be accorded judicially construed to have a certain meaning, or
their natural and general significance93 has a well-known legal meaning97
Exception: Legislative intent or qualification to the
- General term or phrase should not be reduced
into parts and one part distinguished from the contrary Examples:
other so as to justify exemptions from the 1. “Proper action” is an ordinary suit which
operation of law94 prosecutes another for the enforcement of a
right98
- Corollary to this is where the law does not
make any exceptions, the court 2. “Acquitted” is the finding of not guilty based on
may not except something from merit, hence, different from dismissal
it Maxim: ubi lex non distinguit (insufficiency of evidence)99
nec nos distinguire debemos (where the law

92 96
Geotina v. CA San Miguel Corp. v. Municipal Council of Mandaue

93 97
Lo Cham v. Ocampo Krivenko v. Register of Deeds

94SSS v. City of Bacolod 98 Manila Herald Publishing Co, Inc. v. Ramos


95Asiatic Petroleum Co. v. Commissioner of
Internal Revenue 99 Manlayaon v. Lising

JUSTIN SUCGANG | BLOCK 2 ATTY. WIGBERTO TANADA, JR.


3. “Previously convicted” refers to Gachon v. Devera, Jr.
the date of conviction and not date of A complaint of forcible entry was filed by the private
commission respondents. Claiming to be an oversight, they filed a
of crime100 petition for extension of time to file an answer with the
MTCC but was denied since it was a prohibited pleading.
After the reglementary period, they filed for admission of
Identical terms answer and an amended answer. The MTCC denied the
- The general rule is words or phrases repeatedly motions and considered the case submitted for resolution. It
used in a statute will bear the same and consistent resolved in favour of private respondents.
meaning throughout the Held: All other provisions in the Rules of Summary Procedure
whole statute101 When used the word “shall” in matters pertinent to pleadings and
applied: answers. The policy beyond this is to prevent needless
delays and an orderly and speedy discharge of judicial
1. Particularly applies where words appear so
business. Hence, respondent judge may render a judgment,
near each other physically motu proprio.
2. The word has a technical meaning defined in
the statute Dictated by context
Exception: Unless a different intention appears or is
- The context dictates how words should be
clearly expressed
construed. o Broad sense to a word of
Presumption: Words used in a statute is used in
ordinarily
the same sense throughout the whole law
limited meaning (e.g. riparian refers not only
those living near the river but have a water
Qualified by purpose of statute frontage)103
- The purpose is that which induced the legislature
o Limited meaning of a word of broad
to enact the statute
signification (e.g. overthrow in the
- Hence, the court should adopt that interpretation
AntiSubversion Act is limited to those by force
that accords best with the manifest purpose of the
or violence)104
statute or promotes or realizes its object102
o Generic in one part, limited in another (e.g.
- Construction should be rejected if it negates the
owner may be real or the manager)105 Maxim: verba
purpose of the law
accipienda sunt secundum materiam (a word is to
be understood in the context in which it is used)
Construed in relation to other provisions
- Words and phrases must be interpreted in relation
Disjunctive and conjunctive words
to other provisions, and not in isolation
Disjunctive words signify disassociation and
- Variation of construction as a whole
independence of one thing from each other
enumerated (e.g. “or”). Exception: When the
Adasa v. Abalos
spirit or context of the law warrants, it may be
Petitioner was charged of receiving and encashing a check
given a meaning as that of: (1) “and;” (2) “that is
issued in the name of respondent without his consent and
to say” (given that which precedes it the same
knowledge. After finding a probable cause, he was charged
with by city prosecutor. In his arraignment, he pleaded not significance as that which follows it); or (3)
guilty. Dissatisfied with the finding of the latter, he filed a “successively” (in the order of).
petition for review before the DOJ, of which the DOJ ordered
the prosecutor to withdraw said case. Respondent filed a Conjunctive words means “together with,”
motion for reconsideration arguing that the DOJ should “joined with,” “along or together with,” “added to
dismiss outright the petition since there has been an or linked to” used to conjoin word with word,
arraignment. phrase with phrase, clause with clause.
Held: The word “may” in the phrase “may reverse, affirm or Exception: When the literal interpretation would
modify” in Sec. 12 of the DOJ circular should be read as
pervert the plain intention of the legislators as
“shall” in relation to Sec. 7 which positively prohibits a giving
gleaned from the context or extraneous factors, it
due course if there has already been an
arraignment. Since there are no may: (1) mean “or;” (2) restrict the meaning of a
irreconcilable differences or broad word (if there is a broad and the other
contradiction between said restrictive).
provisions.

100
Rura v. Lopena
104 People v. Ferrer
101
Krivenko v. Register of Deeds
105 People v. Nazario
102 Luzon Stevedoring Co. v. Natividad
103 Santulan v. Executive Secretary

JUSTIN SUCGANG | BLOCK 2 ATTY. WIGBERTO TANADA, JR.


And/or means the effect shall be Ejusdem generis
given to both the conjunctive and disjunctive, that - While general words are accorded their generic
is taken according to the one that will best sense, as a rule, they will not be given such
effectuate the purpose intended by the legislature meaning if they are used in association with
Reason: To avoid construction that excludes the specific words.
combination of several alternatives or the efficacy - When a general word follows an enumeration of
of one standing alone. particular and specific words of the same class, the
general word is to be restricted to persons, things
or cases of the same kind as those specifically
Using associated words mentioned. Presumption: Had the legislature
intended the general words to be used in their
1. Noscitur a sociis generic and unrestricted sense, it would not have
2. Ejusdem generis enumerated specific words since the minds of the
3. Expressio unius est exlclusio alterius legislators are addressed to the particularization.
Requisites:
(negative-opposite doctrine)
1. Enumeration of particular and specific words,
4. Casus omissus
followed by a general word
5. Doctrine of last antecedent (reddendo singular
2. The specific words constitute a class or of the
singulis)
same kind
Noscitur a sociis 3. Enumeration is not exhaustive or be an
example
- Where the law does not define a word, it will be
construed as having a meaning similar to 4. No indication of legislative intent to give the
general words or phrases a broader
that of words associated or accompanied by
meaning Limitations:
it 106

1. The specific words do not constitute a readily


- Where most of the word in an enumeration of discernible class and are patently not of the
words in a statute are used in their generic and same kind
ordinary sense, the rest of the words should 2. There is the regulatory intent to give the
similarly be construed general phrase a broader meaning
3. The most important is that the statutory
People v. Delantar provision is not itself restrictive
Defendant Delantar was charged because of facilitating and
4. Where the law is clear and free from ambiguity
inducing his stepdaughter to sexual servitude. Because of
their relationship, it was claimed that the crime was
attended with an aggravating circumstance of relationship. Vera v. Cuevas
Held: His relationship with the victim is not included in those Petitioner, the commissioner of internal revenue, ordered
enumerated. It was not proven that he was the real or the withdrawal of the private respondents’ filled milk
biological father. At the most, he was said to be the products from the market which do not bear the inscription
guardian. But the word “guardian” envisaged by the law is required by Sec. 169 of the Tax Code. Held: The broad
that of having a legal relationship considering the other phrase “all milk” in said provisions should be understood
words in the list. within the meaning of skimmed milk since the headnote
(skimmed milk) and the text (condensed skimmed milk) talk
about this. Hence, the inscription is not required upon this
Carandang v. Santiago
kind of milk.
Petitioner survived the attack of respondent Santiago. As a
result, respondent was charged with frustrated homicide.
Carandang filed a civil case to recover damages against Expressio unius est exlclusio alterius
respondents and his parents. Respondents argued that - Express mention of one person, thing or
frustrated homicide was not included as basis for a civil case consequence implies the exclusion of all the others
under Art. 33 invoked by the petitioner. - Also known as negative-opposite doctrine
Held: The term “physical injuries” - Canon of restrictive interpretation based on the
under Art. 33 should not be
rules of logic and the natural workings of the
construed as a specific crime falling
human mind (particularization)
under the Revised Penal Code.
Other words associating it- defamation and fraud- do not - Opposite of doctrine of necessary implication
have a specific definition and provisions in the RPC. Hence, it - Generally used in the construction of statutes
should be understood in its generic sense, any bodily injury. granting powers, creating rights and remedies,
restricting common rights, and imposing penalties
and forfeitures, as well as those strictly constured

106 Co Kim Chan v. Valdez Tan Keh


JUSTIN SUCGANG | BLOCK 2 ATTY. WIGBERTO TANADA, JR.
Presumption: The legislature would not Pimentel III v. COMELEC
have made specified enumerations in the statute had Petitioner and private respondent Zubiri were fighting for
the intention been not to restrict its meaning and the 12th place for the senatorial position. Pimentel assailed
confine its terms to those expressly mentioned. the canvassing of the Municipal COCs by the Special
Variations: Provincial Board of Canvassers (SPBOC) for Maguindanao,
created because of the alleged irregularity in the PBOC
1. What is expressed puts an end to that which is chaired by Lintang Bedol. Held: As a general rule, pre-
implied (i.e. if the statute is expressly limited, it proclamation cases for national positions are prohibited
may not be extended to other matters) under Sec. 15 of RA 7166 to avoid delay in the proclamation
2. A thing that is not excepted must be regarded of the winner. But it allows an exception if the authenticity
as coming within the purview of the general of COCs questioned are now lodged in the COMELEC en banc
rule (exceptio firmat regulam or the Congress sitting as National Board of Canvassers.
in casibus no exeptis) Limitations: Because the pre-proclamation cases at the provincial level is
not mentioned in the exceptions, it is still prohibited.
1. Not applicable where words are used by
example only or to remove doubts
E.B. Villarosa & Partner Co, Ltd. v. Benito Petitioner agreed
2. When the enumeration was not intended to be to develop certain parcel of lands owned by respondents in
exclusive Cagayan de Oro, and agreed that litigations that may arise
3. If there is no reason why other persons or shall be tried in Makati. Private respondent filed a case
things not so enumerated should not have against petitioner for breach of contract for failure to make
been included and manifest injustice will follow any developments. Summons was served upon the
by non-inclusion (violation of equal protection defendant through its branch manager. They moved to
clause) dismiss the case on the ground of improper submission of
summons.
4. When it defeats the plainly indicated purpose
Held: The Rules of Court initially provided that service of
of the legislature
summons may be made to a manager, secretary, cashier,
5. If it leads to inconvenience, hardship and injury agent, etc. until it was amended by the Rules of Civil
to public service Procedure saying that only a “general manager,” “corporate
secretary,” “treasurer” may be summoned; the word
Malinias v. COMELEC “agent” was deleted from the list. Hence, the new law
Petitioners, candidate for governor and congressman, strictly provided who may be summoned to ensure that the
charged private respondents, their election rivals. They corporation will receive prompt and proper notice of actions
claimed that private respondents prohibited them to attend against it.
the canvassing under Sec. 25 of RA 6646 and even entered Because of this, the trial court did not acquire jurisdiction.
the canvassing room violating Sec. 232 of BP 881.
Held: Both RA 6646 and BP 881 specifies all election offenses Primero v. CA
in Sec. 27 and Sec. 261 and 262, respectively. The alleged Orlando Primero was charged and convicted of acts of
violations are not included in said enumeration of criminal lasciviousness with illegal possession of deadly weapon, a
election offenses. At the most, they highlight the recognize bayonet.
rights of a political party and candidate. This is more true Held: Where there is no reason why a thing not enumerated
since criminal offenses should be strictly interpreted in should not be included that will lead to manifest injustice,
favour of the accused. But it does not mean that these are expressio unius est exclusio alterius cannot be applied.
without sanctions because administrative penalties may be Although a bayonet is not expressly mentioned as “bladed,
imposed. pointed or blunt weapon” under PD 9, it cannot be
convincingly argued that it is not a bladed, pointed or blunt
People v. Lopez weapon. If a fan knife, balisong or club, less deadly than a
Judge Eusebio Lopez was charged for being biased in the trial bayonet are prohibited under the law, there is no reason
of Guillermo Francisco, who was accused of treason for why a more dangerous weapon such as a bayonet should be
collaborating with the Imperial Japanese force. He dissented exempted.
to the judgment made by his two associate justices in their
division- he justified the actions of the alleged political Casus omissus pro omisso habendus est
collaborators and even called them patriots. As a result, he
- A person, object or thing omitted from an
was asked to be disqualified by the prosecution. The other
enumeration must be held to have been omitted
associate justices resolved to
disqualify him. Held: There is no intentionally
law effectively disqualifying a judge for - Applies only if and when the omission has been
partiality. Sec. 1 of Rule 124 does not include failure to clearly established
administer impartially as one of the grounds of Exception: When the legislature did not intend to
disqualification. If this be the case, then many justices will be exclude the person, thing, or object from the
disqualified since human nature tells us that everyone has enumeration
his own biases. And even if it may be a ground, only the
challenged judge himself may determine whether he is fit to
Doctrine of last antecedent
render a fair judgment.
- As a general rule, qualifying words restrict or
modify only the words or phrases to which they are

JUSTIN SUCGANG | BLOCK 2 ATTY. WIGBERTO TANADA, JR.


immediately associated, not those applicable)
that are distantly or remotely located107
- The use of a comma (,) to separate an antecedent Amadora v. CA
from the rest exerts a dominant influence in the The son of petitioners was shot by a classmate inside the
application of this doctrine (i.e. qualifying effect or school’s auditorium while doing a physics report. They filed a
civil action for damages under Art. 2180 of the Civil Code
a modifying word will be confined to its immediate
against the school, principal, the dean of the boys and the
antecedent if the latter is separated by a comma
physics teacher.
from the other antecedents)108 Held: Teachers should apply to pupils and students
Maxim: ad proximum antecedens fiat relation nisi (academic), heads of arts and trades to apprentices. This is
impediatur sententia (relative words refer to the because of the closer tutelage found in heads of arts and
nearest antecedent, unless the context otherwise trades than the heads in academic institutions, in which the
requires) teachers practice close supervision.
Exceptions:
1. Where the intention of the law is to apply the Provisos, Exceptions and Saving Clauses
phrase to all antecedents embraced in the
provision, the same should be made extensive 1. Provisos
to the whole. *Slight indication of this intent is 2. Exceptions
sufficient to extend the relative term109 3. Saving clause
2. The intention is not to qualify the antecedent
at all.110 Provisos
- Limit the application of the enacting clause,
Cadayona v. CA section, or provision of a statute, or to except
Petitioner was suspended by the Civil Service Commission. something from it, or qualify or restrain its
As a result, he filed a motion for reconsideration with the generality, or to exclude some other ground of
CA. The latter dismissed it because three annexes were not exclude some possible ground of
certified true copies. Held: The doctrine of last antecedent
misrepresentation
cannot be applied qualifying “other supporting papers” with
“certified true copies.” Only the judgment, final order or
Exception: If there is a contrary legislative intent
resolution are required to be certified true copies. The court
cited several provisions showing that only the three need be *Commonly found at the end of a section,
certified true copies. To rule otherwise would be too harsh provision, and introduced by the following:
and would not promote a just, speedy and underlying 1. Provided
disposition of the case. 2. But nothing herein

Florentino and Zandueta v. PNB


*But the use of “provided” does not necessarily
Petitioners filed a mandamus to compel PNB to accept their
backpay certificate to pay their debts. However, PNB refused
make the provision a proviso. What determines
arguing that there was a qualification of “who may be willing whether a clause is a proviso is not is its substance
to accept the same as settlement” in the statutory provision. then its form.
Held: Said qualification should refer to the last antecedent
“any association or corporation organized under Philippine Roles:
laws.” The purpose of the statute is the recognition of the 1. Limit or restrain the general language
government of the contribution of veterans to the resistance
2. Enlarge, restrict or limit a phrase of limited
movement. Hence, its agencies must accept said certificates.
import had there been no proviso qualifying it.
To impose this on private corporations would tantamount to
deprivation of property because of the lower rate it has. 3. Give additional legislation
4. Qualifies or modifies the phrase immediately
Redendo singal singulis (referring each to each) preceding it.
- Referring each phrase to its appropriate object
- Let each be put in its proper *If there is a repugnancy between a proviso
place and the main provision, the first step is
harmonizing the two. If there is an
- Words should be taken
irreconcilable conflict, that which is located in a
distributively
later portion of the statute prevails since it is
(Each word is to be applied to the subject
the latest expression of legislative intent.
to which it appears by context most
appropriately related and most

107 Felipe v. De la Cruz 109 Ochate v. Deling

108 110
Nera v. Garcia Heras Teehankee v. Director of Prisons

JUSTIN SUCGANG | BLOCK 2 ATTY. WIGBERTO TANADA, JR.


Exceptions
- Clause which exempts or removes something from
the operation of a statute by express words111112
- Words used: except, unless, otherwise, shall not
apply
- But it need not always be introduced by these
words.
- Confirms the general rule
- Express mention of exceptions excludes other
exceptions; and conversely, those which are not
within the enumerated exceptions are deemed
included in the general rule Maxim: exceptio firmat
regulam in casibus non exceptis (a thing that is not
excepted must be regarded as coming within the
purview of the
general rule)

Differences of an exception to a proviso120


Exception Proviso
Exempts something Defeats the operation
absolutely from the conditionally
STATUTE CONSTRUED AS WHOLE AND IN
operation of law, by
express words
RELATION TO OTHER STATUTES
Takes out of the statute Avoids something by
Construction of a statute as a whole
something that would way of defeasance or
be a part of it otherwise excuse
A statute is passed as a whole and not in parts or
sections and is animated by one general purpose and
Part of the enactment When the enactment is
intent. Hence, it should be construed as a harmonious
itself modified by engrafting
a new provision, by way whole.
Maxim: ut res magis valeat quam pereat (that
of amendment
construction is to be sought which gives effect to the
It is said that one of the function of a proviso is to whole of the statute)
except something from an enacting clause, in this
sense, they are similar *Unsafe way of construction: Dividing a statute by a
process of etymological dissertation, into separate
Saving clause words, and then apply each, separated from its
- Operates to except from the effect of the law what context, some particular definitions given by
it provides113 or to save something which would lexicographers, and then reconstruct the
otherwise be lost114 statute upon the basis of these definitions.115
- Usually used to except or save something from the
effect of a repeal Importance in construction
- In construing it, the intent of the legislature shall The intent of the statute is ascertained from it
be considered or the purpose which the legislature taken as a whole. And this purpose (intent) controls its
had in mind in enacting said law construction- how each word and phrases may be
given meaning.

Intent is ascertained from statute as a whole


- The intent or meaning of it should be
ascertained from the statute taken as a whole
and not from isolated parts of it.

111 Pendon v. Diasnes


114 Ibanez de Aldecoa v. Hongkong & Shanghai Bank
112 115 Sotto v. Sotto
Rowell v. Janurin

113 Bautista v. Fule


JUSTIN SUCGANG | BLOCK 2 ATTY. WIGBERTO TANADA, JR.
- A statute should be construed months later, 3 creditors filed for an
with reference to every other part and every involuntary insolvency of the spouses. Held: The lien on the
word and phrase in connection with its context. disputed properties overrode the insolvency proceeding and
Maxim: optima statuti interpretatrix est ipsum was not dissolved. Sec. 32 provided a cut-off period where
levy on attachment shall be dissolved (1 month and 30 days
statutum (the best interpreter of a statute is
for judgments entered in actions commenced prior to the
the statute itself)
proceeding). Sec. 79, where respondents relied, talks about
the right of the plaintiff and does not dissolve the
Purpose or context as controlling guide attachment. Construction should give effect to the every
- The meaning attached to any word or phrase in word of the whole statute.
a statute is usually ascertained from the
context, the nature of the subject treated and Aquino v. Quezon City
the purpose or This consolidated two cases involving the non-payment of
intention of the body which enacted it real property tax. In the first case, Spouses Aquino withheld
Construction of a statute as a whole the payment of real property tax as a protest to the
government of Marcos. In the second, Solomon Torrado paid
Because a statute is enacted as a whole and not in
taxes on the improvements but not on the land itself
parts, every part is as important as the other. Hence, in because the Treasurer cannot find the index card for the
order to properly and intelligently construe a provision land. Because of non-payment, the QC government sold the
or section of a statute, understand its meaning and delinquent properties. The petitioners contend that they
scope, and apply it to an actual case, the courts should were not notified of the transactions.
consider the whole act itself.116 (i.e. a provision unclear Held: The Real Property Tax Code should be construed as a
by itself may be made clear by reading and construing whole. Although it required a Notice of Delinquency in Secs.
it in relation to the whole statute) Presumption: The 65 and 67, the respondents have complied with it
legislature has enacted a statute whose provisions are satisfactorily. Posting and publication is not indispensable so
in harmony and consistent with each other and that long as there was personal service. On the other hand, sec.
73 gives an option to the Treasurer where to send the
conflicting interpretation in the same statute are never
notices: (a) address shown in tax rolls, (b) residence, if
supposed or regarded. known to the treasurer or barrio captain. Torrado indicated
Exception: If one part of a statute cannot be reconciled in his address, “Butuan City,” and the Treasurer twice mailed
or harmonized with another without nullifying one in in this address (notice of delinquency and notice of sold
favour of another, the court should construe it by property). It is immaterial if they have received the notice so
choosing the one which will long as there was strict compliance to the provisions of the
best effectuate the legislative intent law by the Treasurer.

How done:117 Aisporna v. CA


Mapalad Aisporna was charged with a violation of the
1. Take the thought conveyed by the statute as a
Insurance Act for selling insurance policies without license.
whole She claimed that she was just helping her husband, the
2. Construe the constituent parts together registered insurance agent. Held: The statute must be
3. Ascertain the legislative intent from the whole construed as a whole. Sec. 189(1), though prohibiting the
act any person to act as agent without authorization from the
4. Consider each and every provision in the light Insurance Commissioner, Sec. 189(2) defined who an agent
is- selling policies for compensation. This definition should
of its general purpose
be applied in relation to other paragraphs. Hence, without
5. Endeavor to make every part effective, proof of compensation, Aisporna may not be held liable.
harmonious and sensible
Sajonas v. CA
Apparently conflicting provisions reconciled Spouses Sajonas decided to buy on instalment from Spouses
- All provisions, even if apparently contradicting, Uychocdes a piece of land. A deed of absolute sale with
should be allowed to stand and given effect by adverse claim was issued to them. But there was a
reconciling them compromise agreement between the vendor spouses and
- The courts should endeavour the Spouses Pilares (their creditor). A notice of levy of
to reconcile statutes instead of execution was inscribed in the title. Held: Sec. 70 of PD1529
provided that an adverse claim shall be effective for 30 days
declaring outright the
from the date of registration, this is to protect the interest of
invalidity of one against the
a person over a piece of land. This will not be automatically
other cancelled after the lapse of said time, “30 days.” Hence, it
will continue until cancellation.
Radiola-Toshiba v. IAC
A levy on attachment was done in favour of the petitioners
on the real properties of private respondent spouses. Four

116 117
Maddumba v. Ozaeta Republic v. Reyes

JUSTIN SUCGANG | BLOCK 2 ATTY. WIGBERTO TANADA, JR.


Special and general provisions in a provision said, “hereby authorized,” which
statute means Congress only empowered the City. To construe that
- Where there is a particular or special provision the City does not need legislative permission to sell land is to
render the said phrase superfluous.
and a general provision in the same statute, the
former is construed as
an exception to the latter118 Statute and its amendments construed together
- Changes made by the legislature, in the form of
amendments, should be given
Construction as not to render provision nugatory
effect together with the other parts of the
- A portion of a statute should not be
amended act
construed to destroy the other Maxim:
Presumption: It is not presumed that the
interpretatio fienda est ut res magis valeat quam
legislature, in making such changes, was indulging
pereat (a law should be interpreted with a view
in mere semantic exercise. There must be some
to upholding rather than destroying it)
purpose in making them.
Paras v. COMELEC
Held: A statute should be construed in relation with other Construction in relation to the Constitution
provisions. Sec. 74(b) must be construed in relation to
paragraph A, which is the time when a recall may take place As the fundamental law of the land, all statutes
(i.e. second year of the term). should be subservient to the Constitution. Hence, any
statute should be construed in harmony with, and not
Construction as to give life to the law in violation of
- Laws must receive sensible interpretation to it.122
promote the ends of which they are enacted.119 Presumption: The legislature in enacting a law is
- They should be given reasonable and practical presumed to have adhered to the constitutional
constructions will give life to them120 limitations. It was the intention of the legislature to
enact a valid, sensible, and just law.
- It should not be construed as to allow the doing
of an act prohibited by law121 Presumption: The
Construction to avoid conflict with the constitution
legislature did not to a vain
A statute should not be construed in such a way
thing in the enactment of a statute
that will give rise to a constitutional doubt.
- If a statute is reasonably susceptible of two
Construction to avoid surplusage
constructions, one constitutional and one
- The statute should be construed as to make no
unconstitutional, that construction in favour of its
part of it a useless surplusage
constitutionality shall be avoided and the
- All efforts should be exerted to give some construction that will render it invalid rejected.123
meaning to every word or phrase in a statute
Presumption: The legislature is presumed to have - Every intendment of the law should lean towards
used the word or phrase for a purpose, and not to its validity, and the court should favour that
insert a provision which is unnecessary. construction which gives it the greater chance of
surviving the test of constitutionality.124
Manila Lodge No. 761 v. CA - Courts may be even justified in disregarding the
The Philippine Commission enacted Act No. 1360 to reclaim more usual and apparent import of the language
a certain portion of Manila Bay. This gave ownership to the used in the statute and in straining the ordinary
City of Manila but setting aside a portion in the northern meaning of words to avert any collusion or
part where a hotel shall be built. The City of Manila sold it to repugnancy between what the statute provides
BPOE, the latter selling it to Manila Lodge. BPOE sold it again and what the Constitution
to TDC and petitioned to cancel the right of Manila to
requires.125
repurchase the land. The trial court and the CA resolved in
favour of the City of Manila and held that the contested land
is a public park or plaza. Held: The City Paras v. COMELEC
of Manila cannot sell the land Held: A statute should be construed in relation to the
because it is a public park (several Constitution. To construe SK election as a local election is to
arguments were given to support render the Art. X Sec. 3 of the Constitution nugatory. This
this). This needs authorization from Congress when a

118 122 PLDT v. Commissioner of Internal Revenue


Lichauco & Co. v. Apostol

119 Lo Cham v. Ocampo 123 Alba v. Evangelista

120 Arabay, Inc. v CFI Zamboanga 124 San Miguel Corp. v. Avelino

121 Uy Ha v. City Mayor of Manila 125


Mutuc v. COMELEC

JUSTIN SUCGANG | BLOCK 2 ATTY. WIGBERTO TANADA, JR.


would hinder the system of recall granted by amendment or repeal and in favour of
the Constitution. harmonization of all laws on the subject
matter127 Exceptions:
In re: Guarina 1. Unless there is an irreconcilable repugnancy
Mario Guarina wanted to be admitted to the Bar without between them and harmonization or
taking an examination since he is a fiscal of Batanes. He
reconciliation is not possible
relied on Act No. 1597 Sec. 2 stating that provincial fiscals
may be licensed to practice law without taking the exam. 2. The new law evidently intended to supersede
Held: Said provision should be construed in relation to the all prior acts on the matter to comprise itself
organic act (Philippine Bill of 1902). PB of 1902 prohibited the sole and complete system of legislation on
Congress from depriving jurisdiction from the court. Since the subject If harmonization is impossible:
prior to PB of 1902, the Organic Act (Act No. 136) and the - If there has been an implied repeal, the latter
Code of Civil Procedure (Act No. 190) granted the Court the statute should be construed as to modify the
jurisdiction to deny admission to the Bar, the word “may” prior law no further than may be necessary to
should be construed as permissive so as not to decrease the effect the specific purpose of the latter
discretion of the Court regarding admission to the Bar.
enactment128
Asociacion de Agricultores de Talisay-Silay v. TalisaySIlay - If cannot be possibly harmonized, the earlier
Milling Co. one must yield to the later one, it being the
RA 809 was enacted, allowing the increase of shares of latest expression of legislative
planters and labourers in the income. Respondents disagree will129 Phrases:
stating that there was a contract already between them and - Statutes must be construed not only to be
the planters. Held: RA 809 is not unconstitutional since it is
consistent with itself but also to harmonize
in keeping with the social justice principle of the Constitution
with other laws on the same subject matter, as
and a police power measure for the promotion of labor
conditions. It seeks to reduce the inequality received by the to form a complete, coherent, intelligible, and
Central and the labourers. uniform system of jurisprudence130
- The best method of interpretation is that which
Construction in relation to other statutes makes laws consistent with each other
- Two or more statutes on the same subject were
Statutes in pari materia enacted at different times and under dissimilar
- Two or more statutes relate to the same circumstances or conditions, their
specific subject matter. interpretation should be in accordance with the
- Relate to the same person or thing, or have the circumstance or conditions peculiar to each
same purpose or object, or cover the same (distinguire tempora et concordabis jura or
specific or particular subject matter126 distinguish
Presumption: Laws are consistent with each times and you will harmonize laws)131
other. Whenever a legislature enacts a law, it
has in mind the previous statutes relating to City of Naga v. Agna
the same subject matter, and in the absence of Respondents wanted to claim refund for the tax they paid to
Naga after the enactment of Ordinance No. 360, changing
any express repeal or amendment, the new
the scheme of taxation. They contend that the ordinance did
statute is deemed enacted in accord with the
not take effect in 1970 but a year after its approval.
legislative policy embodied in those prior Held: The statutes in pari materia, talk about the enactment
statutes. Maxim: interpretare et concordare and effectivity of tax ordinances. Since it involved only a
leges legibus est optimus interpretandi modus change in taxation, it fell under Sec. 2309 of the Revised
(every statute must be so construed and Administrative Code. But if it creates an entirely new tax,
harmonized with other statutes as to form a Sec. 2 of RA 2264 (Local Autonomy Act) shall govern
uniform system of jurisprudence) (effective 15 days after the passage).

Construing statutes in pari materia Ask the Commissioner of Internal Revenue v. SC Johnson Respondent
question: Does the later act was obliged to pay their parent company royalties based on
impliedly amended or repealed a percentage of net sales. It is also subjected to 25%
withholding tax on royalty payments. They filed for refund to
the earlier statute?
the BIR for overpaying arguing that a preferential rate of
General rule: A statute will not be construed as 10% should apply to them. Respondent court ruled in favour
repealing a prior act on the same subject. In case of
doubt, it will be resolved against implied

126 City of Naga v. Agna


127 City of Naga v. Agna 130 Republic v. Asuncion

128 Lacson v. Roque 131 Commissioner of Customs v. Superior Gas & Equipment Co.

129 Id at 135
JUSTIN SUCGANG | BLOCK 2 ATTY. WIGBERTO TANADA, JR.
of SC Johnson saying that the RP-US Tax Treaty
is related to the RP-West Germany Tax Treaty. Lichauco & Co. v. Apostol
Held: The two treaties are not in pari materia. This is Petitioner wanted to import draft and bovine cattle for the
because the German Tax Treaty expressly allows crediting manufacture of serum from Pnom-Pehn. But respondent did
against German income and corporation tax unlike with that not admit the cattles. Held: Sec. 1770 of the Administrative
of the US. The US does not have a similar matching credit Code has not been repealed by Act No. 3052. Unless there is
scheme and no commitment on the grantee to give some an irreconcilable repugnancy, there can be no implied
form of tax relief to the grantor. Thus, the “most favoured repeal. Sec. 1770 (AC) deals with a special contingency (any
nation” preferential tax rate should not apply. animal from which the Department Head declares
that a dangerous communicable disease prevails in a foreign
Statutes in pari materia country) not subject to Act No. 3052, amending sec. 1762
1. General and special statutes (permitting a draft or bovine cattle for the manufacture of
serum).
2. Reference statutes
3. Supplemental statutes
Butuan Sawmill v. City of Butuan
4. Reenacted statutes The City of Butuan, acting on its charter RA 523 in 1950,
5. Adopted statutes passed an ordinance taxing businesses engaging electric
light, heat and power. Butuan Sawmill, which was given a
1. General and special statutes legislative franchise by virtue of RA 399 in 1949, disputed the
constitutionality of the taxing ordinance because it impairs
the obligation of contract. Held: Although the charter of
General statute- applies to all people of the state
Butuan was approved later than the grant of franchise, the
or to all of a particular class of persons in the state
latter was special and the former general. The charter gave
with equal force132 taxing power to Butuan but businesses of electric light, heart
- embraces a class of subjects or and power should serve as exemptions. Also, the Local
places and does not omit any subject or place Autonomy Act did not authorize Butuan to tax the franchised
naturally belonging to such business since it would result to double taxation because a
class133 franchise tax is already paid.

Special statute- relates to a particular person or 2. Reference statutes


things of a class or to a particular portion - Refers to other statutes and makes them applicable
or section of the state only134 to the subject of legislation.
- Incorporation in a statute of another statute by
General rule: The special must prevail since it evinces reference
the legislative intent more clearly than that of a - Used to avoid encumbering the statute books of
general statute and must be taken as intended to unnecessary repetition
constitute an exception to the - Recognized as an approved method of legislation in
general act135 the absence of constitutional restrictions
- Adoption by reference of a statute previously
*The circumstance that the special law is passed before repealed revives that statute but does not include
or after the general act does not change subsequent changes or modification
the principle
People v. Martin
Reason: The legislature has its attention directed to the Hermin Arceo and Simeona Martin were charged with
possession of counterfeit science stamps. The CFI dismissed
special facts and circumstances which the special act is
it since RA 5448 did not punish said crime because no
intended to meet. It considers and makes provisions mention was made regarding a science stamp. The
for all the circumstances of a “penalties” mentioned in RA 5448 refers only to the
particular case.136 Exceptions: administrative and not criminal. Held: Since the statutes are
1. Where the legislature clearly intended the later in pari materia, sec. 4(2) of RA 5448 referred to the manner
general enactment to cover the whole subject and and time of collection, and the penalties provided in Sec. 240
to repeal all prior inconsistent of the National Internal Revenue Code (NIRC). This made the
laws provision complete and as effective and fool proof as the
earlier statute, hence, it must be criminal in nature.
2. Where the special law
merely establishes a general rule while the general
law creates a
specific and special rule

132 US v. Serapio
135 Lichauco & Sons v. Apostol
133 Valera v. Tuason
136 Manila Railroad Co. v. Rafferty
134 Id at 140
JUSTIN SUCGANG | BLOCK 2 ATTY. WIGBERTO TANADA, JR.
3. Supplemental statutes Held: Congress did not expand the scope of
- Intended to supply deficiencies in an existing the exemption to include instrumentalities or agencies
statute and to add, to complete, or extend of
the government-owned and controlled corporations.
the statute without changing or modifying its
original text
The reproduction of the source of exemption in Sec.
40(a) of PD 464 (Real Property Tax Code) in the Local
4. Reenacted statutes
Government Code excluded the GOCCs.
- The provisions of an earlier statute are reproduced
in the same or substantially the same words
- Two statutes with a parallel scope, purpose and 5. Adopted statute
terminology should, each in its own field, have a - A statute patterned after, or copied from a
like interpretation, unless in particular instances statute of a foreign country - In construing
there is something peculiar in the question under adopted statutes, the court must take into
consideration, or dissimilar in the terms of the act consideration: o construction of the law by the
relation to it, requiring a different conclusion courts of the country from which it is taken, o
- If a statute has been construed by the court of last law itself o practices under it
resort and the same is re-enacted, the legislature Exception: construction given to the statute
may be regarded as adopting such construction subsequent to its adoption, but it has persuasive
(adoption of contemporaneous construction) effect
- Applies only when the statute is capable of the
construction given to it when the construction has Banas v. CA
become a settled rule of conduct Banas sold a land to Ayala Investment for P2,308,770.
- Reenactment may also be done by reference In the Deed of Sale, it was agreed that he shall be paid
in instalment for four years. On the same day, he
discounted the promissory note with Ayala. He was issued
ESSO Standard Eastern v. Commissioner of Internal Revenue 9 checks all dated with that day drawn against BPI. In
Petitioner asked for refund for overpaid taxes arising from the first year, he reported P461,754 as his
the disallowance of the margin fees paid to the Central Bank
on its profits to its New York head office. They claim that
income, and a uniform income of P230,877 for the
margin fees are taxes, hence, should be deductible from
succeeding years. He was charged for tax evasion but
ESSO’s gross income under Sec.
applied for tax amnesty under PD 1740. He filed for
30(c) of the NIRC.
another tax amnesty under PD 1840. But he still
Held: The margin fee was not a re-enacted statute of a
insisted that he was paid in instalment. Held: To avail of
revenue measure. It is in keeping with the police power of
the tax amnesty, there should be a disclosure of untaxed
the State to discourage imports and encourage exports. It
income. When he discounted the check, he lost his
cannot also be considered as a necessary expense deductible
entitlement of payment through instalment since he
to the net income because it is paid to its head office as a
immediately received cash. The law was adopted from
separate income, and not as an appropriate and helpful in
the US where it held that an
the development of the business. Tax exemptions are strictly
instalment obligation discounted at a bank, it would
construed against the taxpayer.

result to a taxable disposition. Hence, he should have


Mactan Cebu International Airport Authority v. Marcos
MCIAA was created by virtue of RA 6958 and enjoyed tax declared the whole income in that year.
exemption under Sec. 14 of its Charter. The City Treasurer
demanded for payment of real estate tax. It was contended
that, aside from Sec. 14, the Local Government Code
exempted the National Government and its agencies, and
the MCIAA is an instrumentality of the government
performing governmental functions.

STRICT OR LIBERAL narrowest meaning. Nor does it mean that words


CONSTRUCTION should be restricted as not to have their full
meaning.
Strict construction
- Scope of the statute shall not be enlarged or Liberal construction
extended by implication, intendment, or equitable - Equitable construction as will enlarge the letter
consideration beyond the literal meaning of the statute to accomplish its intended
- Close and conservative adherence to the literal or purpose, carry out its intent, and promote
textual interpretation Limitations: It does not justice
mean that the statute should be given its

JUSTIN SUCGANG | BLOCK 2 ATTY. WIGBERTO TANADA, JR.


- Expand the meaning of the d. Statutes prescribing formalities of will
statute to meet cases which are clearly within 4. Statutes concerning the sovereign
the spirit or reason of it a. Statutes authorizing suits against the
Exceptions: If the statute is plain, clear and government
unambiguous, enlargement of the provision is 5. Exceptions and provisos
prohibited. Nor does it mean that the words be forced
out of their natural meaning.
1. Penal statutes
- Those which define crimes, treat of their nature,
*Liberal construction is not engrafting upon a law
and provide for their punishment
something which the Court believes ought to be there.
Liberal construction is a valid exercise of judicial power, - Strictly construed against the State and liberally in
the latter is judicial legislation that is forbidden by the favour of the accused138 Reason:
doctrine of separation of powers. 137 a. The law is tender in favour of rights of an
individual; the object is to establish a certain
*In interpreting the law, the following should be rule by conformity to which mankind would be
taken into account: safe, and the discretion of the court limited.
1. Social justice b. It is not to enable a guilty person to escape
punishment through a technicality but to
- The constitutional mandate on social justice is
provide a precise definition of forbidden acts139
addressed not only to the legislative branch
but also to the two other departments of the General rule: Penal statute shall not be construed to
government make the commission of certain prohibited acts
criminal without regard to the intent of the doer
- If the provision is susceptible of two
Maxim: actus non facit reum nisi mens sit rea (the act
interpretations, they now have to be construed
itself does not make a man guilty unless his intention
to promote and achieve social justice.
were so) actus me invite factus non est meus
2. General welfare or growth of civilization
actus
Maxim: salus populi est suprema lex (public good
(an act done by me against my will is not my act)
is the highest in law) statuta pro public
Exception: If acts are only mala prohibita and the
commodo late
statutes plainly prohibits an act without implying that it
interpretantur (statutes enacted for the public
be done knowingly of wilfully.
good are to be construed liberally)
- Statutes and judicial decisions alike
Limitations to construction in favour of the accused:
come into being out of the same
1. Not to be construed as to defeat the obvious
common roots, the supreme good of
purpose of the legislature
society
2. Only when the law is ambiguous and there is
- Statutes are not isolated from the
doubt in its meaning
drama of life as it unfolds, hence, they
must be interpreted in the light of the
Buenaseda v. Flavier
growth of civilization and varying
Petitioners challenged the power of the Ombudsman to
conditions. issue preventive suspension to employees working in
government offices other than the Office of the
Statutes strictly construed Ombudsman. This is because of the provision, “The
Ombudsman or his Deputy may preventively suspend any officer or
employee under his authority pending an investigation.”
1. Penal statutes
Held: Preventive suspension is not a penalty and just a
2. Statutes in derogation of rights matter of procedure. Hence, it must be liberally construed.
a. Statutes authorizing expropriations To do otherwise is to render the Ombudsman powerless in
b. Statutes imposing taxes and custom his investigation since he can issue preventive suspension
duties only to those working directly under his office.
c. Statutory grounds for
Centeno v. Villalon-Pornillos
removing officials
Petitioners, officers of the Samahang Katandaan ng Nayon
3. Statutes granting ng Tikay, were charged with violation of PD
privileges 1564 or the Solicitation Permit Law by Judge Angeles.
a. Legislative grants to local government They were asking for money to renovate their chapel. Held:
units The court distinguished religious and charitable purposes
b. Naturalization laws and held that solicitations for religious purposes are not
c. Statutes granting tax exemptions covered by said law. Although solicitations for religious
purpose are considered charitable also, not all charitable

137 Tanada v. Yulo 139 People v. Purisima


138 People v. Subido
JUSTIN SUCGANG | BLOCK 2 ATTY. WIGBERTO TANADA, JR.
actions are religious in nature. Hence, penal passage of RA 1608, respondent was ordered
laws should be construed in favour of the accused. to pay taxes for these losses since the tax in alcohol is
attached not as soon as it has existed but on the finished
People v. Salarza, Jr. product. Held: The law should not be given a retroactive
Zareen Smith claimed that she was raped by the defendant effect.
after a drinking session. Held: Accused was acquitted since Hence, it must be strictly construed against the Government.
the elements of rape are not satisfied. It is impossible that It was clear that during said years, the prevailing rule was to
force or intimidation attended the act since there was exempt La Tondena as a result of said losses.
consent when she thought that the one having sex with her
is her lover and passively acceded to the act. Salarza did City of Manila v. Chinese Community of Manila The City of
nothing to mislead her and even told him who he was and Manila asked the CFI in Manila to expropriate the land for
never used force. Zareen was not deprived of reason or the purpose of constructing a public improvement. The
unconscious since she knew that someone was undressing respondents contend that it was one of the owners of the
her. She admitted that she had sex with her boyfriend parcels of land. Held: Expropriation statutes are strictly
many times and knew that he was fond of foreplay and construed against the expropriating authority. This is
doesn’t have sex when drunk. Hence, she should have because of a derogation of the right. The right to ascertain
known that the man whom she was having sex with was upon trial whether the right exists for the exercise of
not her lover. eminent domain, it intended that the courts should inquire
into, and hear proof upon, those questions.
2. Statutes in derogation of rights
Caneda v. CA
- In the exercise of the police power, the legislature
Mateo Caballero, an old widower executed a last will and
may enact laws curtailing or
testament before 3 attesting witnesses. He passed away
restricting rights of the people even before the commencement of the hearing of the
- Because they are in derogation of common or petition. As a result, a legate was appointed as special
general rights, they are strictly construed and administrator. Petitioner contended that they are related to
confined within their scope the testator and challenged the appointment of a special
a. Statutes authorizing expropriations administrator. The lower court rendered a decision stating
the validity of the will. However, petitioners allege that the
- Same as the above but based on the power will did not contain any attestation clause and lacks several
of eminent domain requirement or element.
- Strictly construed against the expropriating Held: The testator himself signed the will and all pages. It is
authority and liberally in favour of the not important that every page must be signed by the
witnesses. But he lacks the element of a will. To be valid, it
property owners b. Statutes imposing taxes
should be executed in accordance with statutory provisions.
and custom
duties
4. Statutes granting privileges
- Power to tax is incident of sovereignty and
is unlimited in range. That is why it is - Viewed with suspicion because grants of advents
regarded that the “power to tax involves created special privileges or monopolies for the
the power to destroy” grantees
- Taxation is a destructive power which - Strict construction requires that those who invoke
interferes with personal and property the grant should strictly comply with the provisions
rights of the people Maxim: privilegia recipient largam
- Strictly construed against the government interpretationem voluntati consonam concedentis
and liberally in favour of the taxpayer (privileges are to be interpreted in accordance to
c. Statutory grounds for removal of officials the will of him who grants them)
- Those referring to suspension or removal a. Legislative grants to local government units
of public officials are to be construed - Grants of public nature and should be
strictly construed strictly against the grantee
- Must be confined within the limits - Gratuitous donation of public money or
prescribed: causes, manner, property which would result in an unfair
conditions advantage to the grantee
Reason: Removal is a drastic b. Naturalization laws
action that would result to - Strictly construed against the applicant
injustice and harm to public interest for citizenship and should be rigidly enforced
and followed Reason: The right of an alien to
Collector of Internal Revenue v. La Tondena Respondent La become a citizen is a statutory and not a
Tondena was in the business of alcohol. In making the natural one. It does not become vested until
Manila Rum, it uses a low test alcohol which it subjects and
he files a petition and establishes by
re-rectifies to further distillation. At first, they were
competent and satisfactory evidence that he
exempted and given an allowance of not exceeding 7% since
there was a loss in the process of distillation. But after the

JUSTIN SUCGANG | BLOCK 2 ATTY. WIGBERTO TANADA, JR.


has all the qualifications and none the private complainant’s right to due
of the disqualifications specified by law.140 process, is erroneous and the Court may remand the case to
the trial court.
c. Statutes granting tax exemptions
- The law frowns against exemption from
4. Statutes concerning the sovereign
taxation
- Restrictive statutes which impose burdens on the
- It must be strictly construed against the
public treasury or which diminish the
taxpayer and liberally in favour of the
taxing authority Reason: Minimize the rights and interest of the government are strictly
different treatment and foster impartiality, construed
fairness, and equality of treatment among - The statutes, no matter how broad, will not cover
taxpayers.141 Exceptions: the sovereign until it is specifically mentioned142
1. Where the provision of the law is clear and - The government does not fall within the terms of
unambiguous any legislation143
2. There is no qualification for the granting of a. Statutes authorizing suits against the
exemption government144
3. Exemptions in favour of government itself
- The sovereign is exempt from suit, in the
or its agencies
absence of its consent to be sued,
d. Statutes prescribing formalities of will is a universal rule
- Strictly construed as to make - To justify a suit against the government,
noncompliance to the statutory there must be a law conferring the right to
requirements shall invalidate the will maintain the action that must be plain and
- This is not a matter of right of the testator positive145 Reasons:
but of the privilege given by the legislature, 1. Classical - There can be no legal right as
hence, the intent of the legislature and not against the authority that makes the law
of the testator on which rights depend
should apply (nullum tempus occurit regi)
2. Practical- To allow the State to be sued
Esso Eastern Oil v. Acting Commissioner of Custom Esso
without its consent is to make it subject to
wanted to refund the amount it paid for special import used
inconvenience and loss of governmental
in its oil retail business, claiming that they use it in their
business operation. efficiency, with a more adverse result
Held: Esso is not entitled to tax exemption since it was found greater than if the
out that the pump parts were not actuallu used in its doctrine is abandoned
business but are being leased to dealers which Esso was tied
up to. For tax exemption to be availed, there must be actual 5. Exceptions and provisions General rule: All
use of that part in business operation. Mere presumption of doubts should be resolved in favor of the general
tax exemption is not favoured. provision rather than the exception

Martinez v. CA
- Exceptions should be strictly but reasonably
Martinez was charged with libel as a result of the alleged construed, and extends only so far as their
derogatory imputations against then VicePresident Laurel in language fairly warrants
his article. After the DOJ Secretary issued an opinion that - Provisos should be strictly construed since the
said article was not libellous, the prosecutor was directed to legislature sets forth in the general enactment its
cause the dismissal of the information filed. Laurel appealed. policy and only those exempted
Martinez moved to dismiss the appeal on the ground that no
by the proviso should be freed from the
appeal lies from the dismissal of a criminal case. Held:
Petition is denied since the procedural recourse of Laurel operation of the statute
was correct. The order of dismissal was a final order, hence
nothing more could be done in tbe lower court. What was
required was the trial judge’s own Statutes liberally construed
judgment and not that of the
opinion of the DOJ secretary. The
1. General social legislation
dismissal order, being a violation of
2. Grant of power to local governments

140 Mo Yuen Tsi v. Republic 143 Id.

141
Resins, Inc. v. Auditor General 144
Sec. 3 Art. XVI (Const). The State may not be sued without its consent.
142
Alliance of Government Workers v.
Minister of Labor and 145 Compana General de Tabacos de Filipinas v. Government
Employment

JUSTIN SUCGANG | BLOCK 2 ATTY. WIGBERTO TANADA, JR.


a. General welfare clause The CA ruled in her favour.
b. Statutes granting taxing power to local Held: Because Hepatitis B was not on the list of occupational
government diseases, the burden of proof lies with private respondent.
Hence, she should have substantiated her claim and not
3. Statutes prescribing prescriptive period to
solely rely on the sweeping rapports from the PNP based on
collect taxes a layman’s report and not on medical findings. Although
4. Statutes imposing penalties for nonpayment of general welfare legislations are construed in favour of the
taxes worker, this holds true only when there is doubt and
5. Election laws ambiguity. The law was clear that causal relation must be
6. Amnesty proclamations backed up by substantial evidence.
7. Statutes prescribing prescriptions of crimes
Sun Life Insurance v. CA
8. Adoption statutes
Felix Lim had a personal accident insurance policy from
9. Veteran and pension laws petitioner. Two months after the issuance of the policy, he
10. Rules of Court died by gunshot. Suicide was ruled out after the testimony
11. Other statutes of his secretary that after pointing the gun at her, he pointed
a. Curative statutes it at his temple after removing the magazine to assure her
that there was no bullet. His wife sued Sun Life and asked to
b. Redemption law
pay her the face value of the policy since the latter claimed
c. Warehouse receipt laws that there was no accident.
d. Probation law Held: As the secretary testified, Lim did not wilfully place his
e. Statutes granting powers to an life at risk. Although Lim was negligent, it should not prevent
agency created by the Constitution his wife from recovering the insurance policy he obtained.
The firing of gun was the additional, unexpected,
independent and unforeseen occurrence that led to his
1. General social legislation
death. Insurance contracts as a rule should be construed in
- These are statutes enacted to implement the social favour of the insured.
justice and protection-to-labor provisions of the
Constitution, and are construed liberally Guerrero v. CA
- The doubt should be resolved in favour of the Spouses Guerrero allowed Apolinario rake care of their cows
persons whom the law intended to benefit and to do chores and harvest in the plantation owned by
them. As such, he shared 1/3 from the proceeds of copra he
- For it is only by liberal construction that the processed and sold to the market. Twice he was refrained
constitutional mandate may be realized Limitation: from harvesting by the Guerreros under threat that his
Holds true only when there is doubt or ambiguity house shall be destroyed. He was ordered reinstated as
in the law and not when it is clear and free from tenant by the CA.
doubt. Held: The Agricultural Tenancy Act and the Land
Reform Act did not repeal the Code of Agrarian Reform.
Santos v. ECC Even if the shared tenancy scheme has been phased out, it
Petitioner’s husband, who worked as welder for the did not intend to make a reversion from tenants to hired
Philippine Navy, died of liver cirrhosis. She then filed a with labourers with no tenurial rights. The sharing of harvests
the GSIS a claim for death benefit, pursuant to PD 626. The taken together with other factors characteristic of a tenant
GSIS denied the claim reasoning out that liver cirrhosis is not strengthens Benitez’s claim.
an occupational disease. If it were so, it should have been a
disease in the lungs, not in the liver. The Employees 2. Grant of power to local governments
Compensation Commissione affirmed the decision. - The trend is from a limited self-government to
Held: Although not listed in Annex A enumerating the full autonomy.146
occupational disease, liver cirrhosis may result from
inhalation of chemicals as a result of gas fumes and heat. a. General welfare clauses - Has
This was his state of work as a welder, and inhalation of two branches:
small amounts, if prolonged would result to this disease. But o The first branch relates to the
even so, the doubt should be resolved in favour of the municipal authority to create
working man. ordinances and regulations as may be
necessary to carry into effect the
GSIS v. CA powers and duties conferred upon the
The surviving spouse of Senior local
Superintendent Jaime legislative bodies by law
Liwanag filed for claim of death benefit. He died of Hepatits
o The second branch authorizes such
B. After being denied by the GSIS, she filed with the CA a
petition where she used as evidence several PNP reports ordinances necessary to provide for
about the death of her husband. the general welfare (healthy, safety,

146
Sec. 25 Art. II (Const). The State shall
ensure the autonomy of local governments.

JUSTIN SUCGANG | BLOCK 2 ATTY. WIGBERTO TANADA, JR.


peace and order, promote These are liberally construed so as not to
prosperity and improve the morals) vitiate the election.
- Construed liberally in favour of the local o Provisions which candidates for office are
governments because the general welfare required to comply are mandatory and failure
clauses are elastic and must be responsive to to comply is fatal
various social conditions o Provisions designed to determine the will
b. Statutes granting taxing power to local of the electorate are liberally construed,
governments147 technical and procedural barriers should not be
- The local governments are now allowed to stand if they constitute an obstacle
empowered to create their own sources of in the choice of elective officials149
revenues, hence, statutes limiting the
taxing power of local governments should 6. Amnesty proclamations
be construed strictly against the National
- Liberally construed in favour of the persons within
Governments and liberally in favour of
the amnesty grant to encourage the return to the
them
fold of law those who have veered away from it
3. Statutes prescribing prescriptive period to collect - The same goes with pardon since the two are
taxes synonymous
- These are beneficial both to the government and
to the taxpayer, since: 7. Statutes prescribing prescription of crimes
o Tax officers would be obliged to act promptly in - Is in the nature of amnesty and should be liberally
making assessments construed in favour of the accused
o Citizens would have a feeling of security - Its existence is a recognition and notification by the
against tax agents who takes advantage and legislature of the fact that time assigned to it
harass taxpayers destroys proofs of guilt
- Liberally construed in a way conducive to bring out
the beneficial purpose of affording 8. Adoption statutes
protection to taxpayers148 - Liberally construed in favour of the child adopted
- This is in consonance in the concept that adoption
4. Statutes imposing penalties for non-payment of statutes, being humane and salutary, hold the
taxes interest and welfare of the child to be of
- Liberally construed in favour of the government paramount consideration150
and strictly interpreted against the taxpayer
- This is to hasten payments of taxes and punish 9. Veterans and pension laws
evasions. If condoned, government services will - Liberally construed in favour of the persons
suffer intended to be benefited
- If possible, it should be interpreted to avoid tax - Veterans laws are expression of gratitude to and
evasions recognition of those who rendered service to the
country by extending monetary benefits151
5. Election laws - Pension and retirement laws intend to provide for
- Reasonable and liberally construed to achieve sustenance and comfort, when the former
the purpose- to safeguard the will of the electorate in employee has no longer the stamina to continue
choosing their representatives - It has three earning his livelihood after devoting the best years
parts: of his life to public service152
o Provisions for the conduct of elections which
election officials are required to follow are 10. Rules of Court
mandatory before the elections - Liberally construed because of being procedural in
and directory after elections. nature.

147 Sec. 5 Art. X (Const). Each local 148 Republic v. Ablaza


government unit shall have the power to 149 Ginete v. Arcange
create its own sources of revenues and to
levy taxes, fees and charges subject to such 150 Malkano v. Agrava
guidelines and limitations as the Congress
may provide, consistent with the basic policy 151
Board of Administrators v. Philippine Veterans Bank
of local autonomy. Such taxes, fees, and
charges shall accrue exclusively to the local 152 Santiago v. COA
governments.

JUSTIN SUCGANG | BLOCK 2 ATTY. WIGBERTO TANADA, JR.


- It should not be interpreted to otherwise be void for want of conformity
sacrifice substantial rights of the litigant at the altar with certain legal requirements.
of technicalities to the consequent impairment - Intended to supply defects. Abridge
of the principles of justice.153 superfluities and curb certain evils,
and are retroactive
Arcilla v. Arcilla b. Redemptive laws
In the annulment of sale, the petitioners did not attend the - Liberally construed to allow the debtor to
pre-trial and filed after the prescribed period. As a result, have his property appliquéd to pay as
the trial court continued with the litigation and allowed the many debtor’s liabilities as possible
presentation of evidence without notice to the other party.
- The doubt should resolve in favour of
Held: Alhtough the Rules of Court are procedural, it should
not be ignored, belittled or dismissed. They are required to exemption from execution or attachment
be followed except for the most persuasive reasons. c. Warehouse receipt laws
- Given liberal construction in favour of the
11. Other statutes holder of such receipts because
a. Curative statutes they play an important role in modern
commerce
- Enacted to cure defects in a prior law or to
validate legal proceedings which would d. Probation law
- Liberally construed by
extending the
benefits to any one
not specifically 162

disqualified
- This is because of the
goal of the probation
to give first-hand

offenders a second chance to

maintain his place in society


e. Statutes granting power to
an agency created by the
Constitution

162 Santos To v. Pano

MANDATORY AND DIRECTORY STATUTES Prohibition; that something be not done, leaving
the person concerned no choice on the matter
Mandatory statutes except to obey154
Positive- Commands that something be done, or - acts executed against the provisions of
performed in a particular way Negative- mandatory or prohibitory laws

153 Cabunilas v. CA
154 Brehn v. Republic
JUSTIN SUCGANG | BLOCK 2 ATTY. WIGBERTO TANADA, JR.
shall be void155 Alcala is qualified by being a former Filipino
citizen who wished to adopt her younger brother Joseph.
But, Art. 185 (FC) orders a mandatory joint adoption for
Directory statutes
spouses. As such, adoption cannot be granted since her
1. permissive or discretionary in nature and merely husband has disqualifications. But, they may still adopt the
outlines the act to be done in such a way that no minor according to the rules on inter-country adoption.
injury can result from ignoring it or that its purpose
can be accomplished in a manner other than that Bunye v. Escareal
prescribed and will have substantially the same Several officials of Muntinlupa were issued a preventive
result suspension by the Sandiganbayan. This is in connection with
2. statute which merely operates to confer discretion the alleged forcible taking of the New Alabang Public
upon a person, namely, to act according to the Market. Defendants claimed that Muntinlupa Government
dictates of his own judgment and conscience, and has a 25-year lease contract with them, and their own
not controlled by the cooperative introduced improvements.
Held: Sec. 13 of the RA 3019 clearly provided that public
judgment and conscience of others
officials charged shall be suspended from office. Even if
there would be massive vacancies in several offices in
How to determine Muntinlupa, a preventive suspension must still be issued.

II. Departure from the language


I. Language used
II. Departure from the language

I. Language used There is no universal rule to distinguish mandatory


from directory statutes. Neither is there an absolute
General rules: test for determining whether a statute is to be
1. Statutes using words of commands (shall, must, considered mandatory or directory. Hence, “shall” may
ought, should) or prohibition (cannot, shall not, be construed as “may and vice versa. To determine the
ought not) connotes compulsion and are construction, the legislative intent must be ascertained
regarded as mandatory and imperative156 from all surrounding circumstance:
2. Negative words or those in form of an
affirmative proposition qualified by the word 1. Entire statute
“only” have the force of 2. Object
exclusionary negation157 3. Purpose
3. This indicates the legislative intent to make the 4. Legislative history
statute mandatory and can rarely be 5. Consequences
directory158
4. Statutes using permissive words (may) or Directory if...
words importing permissiveness are generally a. No substantial rights depend on it
directory.159 b. No injury can result from ignoring it
5. “May” as an auxiliary verb shows opportunity c. The purpose of the legislature can be
or possibility, it implies a accomplished in a manner other than that
possible existence of something160 prescribed and substantially the same results
obtained
Republic v. Toledano and Spouses Clouse Private d. Where the directions of the statute are given
respondents, who were American citizens, wanted to adopt merely with a view to the proper, orderly, and
the minor Alcala. The lower court granted their request, but prompt conduct of business
the Office of the SolicitorGeneral filed a petition claiming
e. Compliance with the statute is a matter of
that the spouses Clouse are disqualified from adopting
because of their citizenship. convenience rather than substance
Held: Art. 184(3) of the Family
Code clearly barred aliens from Mandatory if... 161
adopting when it said “the
a. The doing of an act is required by justice or
following persons may not adopt.” As for Alvin Clouse, he is
not a former Filipino citizen and the minor is neither his
public duty
relative or legitimate child. It may be argued that Evelyn

155 Buyco v. PNB


159 Stanley v. Miller
156 Id at 145
160 Legazpi v. Estrella
157 Valdez v. Tuason
158 McGee v. Republic 161 Diokno v. Rehabilitation Finance Corp.
JUSTIN SUCGANG | BLOCK 2 ATTY. WIGBERTO TANADA, JR.
b. It vests a public body or officer with (he who is first in time is preferred in
power and authority to take such action which right)
concerns the public interests or rights of
individuals San Carlos Milling Co. v. Commissioner of Internal Revenue
c. If it will cause hardship or injustice on the SCMC, by virtue of Sec. 86 of the Tax Code, wanted to have
part of the public its overpaid tax in 1983 be credited against its 1984 tax
dues. The Commissoner of Internal Revenue denied its
d. If it will lead to absurd, impossible or
request and merely treated it as an ordinary claim for
mischievous consequences refund/tax credit subject to further investigation.
e. Non-compliance with what is required will Held: The word “may” in said provision should not be
result in the nullity of the act construed as to give the taxpayer an absolute right to
arbitrarily avail it. It does not mean automatic and
Berces, Sr. v. Guingona, Jr. immediate tax credit, since this would strip off the
Petitioner filed two administrative cases against the mayor government of its power to control taxation. There should
of Tiwi, Albay; of which the latter was convicted. The mayor still be an investigation by the CIR to verify the amount.
appealed to the Office of the President and prayed for “stay
of execution” order. 3. Statutes prescribing jurisdictional requirements
The OP issued a stay for execution. Held: Since implied - Requirements by which courts or tribunals acquire
repeals are not favoured, construction should be to jurisdiction to hear and decide cases must be
harmonize the laws. The phrase in Sec. 68 (LGC), “an appeal strictly complied with (e.g.
shall not prevent a decision from becoming final or publication before a court hear a petition for land
executor,” is held to be discretionary. The OP issued the
registration, written claim of refund to
order so as not to prejudice the public because suspension
would disrupt the service of the mayor.
Commissioner of Internal Revenues before
actions for refund)

a. Statutes prescribing time to take action or to


Mandatory statutes
appeal
- If a decision is adverse to a litigant, it
1. Conferring power
absolutely indispensable to take action or
2. Granting benefits
appeal at the prescribed time in a the
3. Prescribing jurisdictional requirements prescribed manner
a. Prescribing time to take action or to appeal - This is to prevent needless delays and for
b. Prescribing procedural requirements the orderly and speedy discharge of
4. Election laws on conduct of election business
5. Election laws on qualifications and - Failure to do this will leave the appellate
disqualifications court without jurisdiction Maxim:
6. Prescribing qualifications for office reipublicae ut sit finis litium (public interest
7. Assessment of taxes requires an end to a legal controversy)
8. Public auction sale b. Statutes prescribing procedural requirements
- Every act which is jurisdictional, or is an
1. Statutes conferring power essence of the proceeding, or for the
- Confer upon a public body or officer power to protection or benefit of the party affected
performs acts which concern the public interests or
rights of individuals People v. CA
- The power is given not for the benefit of the public Esam Gandi was apprehended in the airport and detained
for possession of Marijuana. After posting bail, he filed a
officer but for third persons to meet the demands
motion for reinvestigation claiming that he was arrested
of rights and to prevent a failure
without a warrant. This petition was denied since he filed it
of justice beyond the 5-day reglementary period. CA ruled that said
period is discretionary only. Held: Rule 112 Sec. 7 is
2. Statutes granting benefits mandatory. He may only exercise his right to reinvestigation
- Requires certain steps to be during the 5-day reglementary period. The reason behind is
taken or certain conditions to to make the process of preliminary investigation speedy.
be met before persons concerned can avail of the Besides, his posting of bail means a waiver to a preliminary
investigation.
benefits conferred by law
- Failure of persons to take the required steps or to
Llenares v. Valdeavella and Zoreta
meet the conditions will prevent him from availing
Llenares filed an action of ejectment claiming that she
of this benefits Maxim: vigilantibus et non owned 12 parcels of land after acquiring from a sheriff’s sale
dormientibus jura subveniunt (law aids the under a writ of execution. The lower court ruled in favour of
vigilant, and not those who slumber on their the defendants since there was insufficiency in the levy of
rights) potior est in tempore, potior est in jure land or in notice of sale. Held: With the admission that the
JUSTIN SUCGANG | BLOCK 2 ATTY. WIGBERTO TANADA, JR.
attempted levy was not made in accordance prevent sacrifice of their property, it is
with the Rules of Civil Procedure, the petition has no merit. mandatory
Proper levy and notice of sale are indispensable to a valid
sale on execution.
8. Statutes concerning public auction sale
- Prescribed steps in public auction of
4. Election laws on conduct of election General rule: All
properties are to be followed strictly Reason: In
provisions governing the conduct of elections and
derogation of the property rights and due process
prescribing steps for the election officials are
mandatory before the elections. If sought to enforce
after election day, they become directory only if this
will deprive innocent voters, without fault on their Directory statutes
part, of their votes
Reason: These steps were adopted to assist the voters 1. Prescribing guidance for officer
in their participation in the affairs of the government 2. Prescribing manner of judicial action
and not to defeat that object. When voters have 3. Rendition of decision within prescribed period
honestly cast their ballots, the same should not be
nullified simply because the officers have failed to do 1. Statutes prescribing guidance for officers
their duty. To make these mandatory would nullify the - Intended to guide public officers in the conduct of
votes affected. Interested people will just be tempted business
and conspire to create irregularities that will result to - So long as these do not limit their power
the vitiation of the election. (jurisdiction) or render its exercise in disregard of
the requisitions ineffectual
5. Election laws on qualification and disqualification - That is why statutes jurisdictional in nature are
- Laws prescribing the time limit to file certificates of mandatory, because it limits the power of the
candidacy and qualifications and disqualifications officer and render their actions invalid (compare
to elective office this to statutes prescribing jurisdictional
- If a candidate failed to comply with said requirements under mandatory statutes)
requirements, he is disqualified to run, even if he Exception: If accompanied by negative words (i.e.
received the highest number of votes.162 Said votes no, shall not, never, etc.)
will be considered void and will not be counted.
2. Statutes prescribing manner of jurisdictional action
6. Statutes prescribing qualifications for office - These are the steps in judicial action followed by
- Eligibility to hold office is a continuing judges in the exercise of their functions
requirements and must exist not only from the - The purpose is to provide an orderly conduct of
beginning of the term but also during the public business.
occupancy of the office - But, the procedure is only secondary in importance
- A person not qualified at the time he assumed to substantive rights, and nonobservance of the
office, or if he loses such qualification during his former should not be permitted to affect the latter.
incumbency will be ousted from office.163
3. Statutes requiring renditions of decision within the
7. Statutes relating to assessment of taxes prescribed period164
- Assessment of taxes, intended to ensure the - A judgment made after the prescribed date of
security of citizens, the equality of taxation, the promulgation is not rendered invalid. But the
certainty as to the nature and amount of each officer who failed to comply with the law may be
other’s tax, are mandatory dealt with administratively, as consequence of his
- But those designed for the methodical and delay.165
systematic modes of proceedings are merely
- Failure of judges to comply merely deprives them
directory
not of their jurisdiction but their right to
Test: If the law is to protect
collect their salaries166
citizens and to

162 Gafor v. COMELEC courts, and three months for all other lower courts. See also Sec. 7. Art.
IX(A). Each Commission shall decide by a majority vote of all its Members,
163Aguila v. Genato any case or matter brought before it within sixty days from the date of its
164Sec. 15(1) Art. VIII (Const). All cases or submission for decision or resolution.
matters filed after the effectivity of this
165 Philippine Association of Free Labor Unions v. Secretary of Labor
Constitution must be decided or resolved
within twenty-four months from date of
166 Dimson v. Elepano
submission for the Supreme Court, and,
unless reduced by the Supreme Court,
twelve months for all lower collegiate
JUSTIN SUCGANG | BLOCK 2 ATTY. WIGBERTO TANADA, JR.
4. Statutes substantive in nature
5. Statutes affecting vested rights
6. Statutes affecting obligations of contract
PROSPECTIVE AND RETROACTIVE STATUTES 7. Repealing and amendatory acts

Prospective statutes
- Operates upon facts or transactions that occur Retroactive statutes
after the statute takes effect
- One that looks and applies to the future167 1. Procedural
- Indicated by the words “hereafter,” “thereafter,” 2. Curative
“from and after the passing of this Act,” “shall have 3. Police power legislations
been made,” “from and 4. Statutes relating to prescription
after,” “shall.” 5. Statutes relating to appeals

Retroactive statutes
- Takes away or impairs vested rights acquired under
existing laws, creates new obligation, imposes a
new duty or attaches a new disability in respect to
transactions already past
- But a statute is not made retroactive because it
draws on antecedent facts for its operation or part
of the requirements for its application is drawn
from a time antedating its passage168
- The constitution does not prohibit the enactment
of a retroactive statute which does not impair
obligations of contract or divest
rights that have become vested

General rule: Statutes are to be construed as having


only a prospective operation169 Exception: The
intendment if the legislature is to give them
retroactive effect is expressly declared
or implied from the language used

Presumption: All laws operate prospectively. And in


case of doubt, it shall be resolved against the
retroactive operations of law. Reason: Law is a rule to
guide actions with no binding effect until it is
enacted. AMENDMENT, REVISION, CODIFICATION AND
REPEAL
Maxims: lex prospicit, non respicit (the law looks
forward, not backward) lex de future, judex *The legislature has the authority to amend, subject to
de praeterito (law provides for the future, the judge constitutional requirements, any existing law.
for the past) nova constitutio futuris formam
imponere debet non praeteritis (a new statute Amendment
should affect the future, not the past) - change or modification, by addition, deletion, or
alteration of a statute
- effected by the enactment of an amendatory act or
Prospective statutes modifying or altering some provisions of the
statute
1. Penal statutes o Express- an amendatory act that specific
2. Ex post facto provisions of a statute are amended as recited
3. Bill of attainder therein

167 169Art. 4 (CC). Laws shall have no retroactive effect, unless the contrary is
Laceste v. Santos
provided.
168 Camacho v. Court of Industrial Relation

JUSTIN SUCGANG | BLOCK 2 ATTY. WIGBERTO TANADA, JR.


o Implied- a part of a prior statute already attached under prior law. This
embracing the same subject as the later act may would be a subversion of the judicial process.175
not be enforced without nullifying the pertinent o This rule also applies to quasi-judicial
provision of the latter. The prior act is deemed bodies.176
amended to the extent of the repugnancy.
- An invalid law does not exist. If a prior statute
which has been amended is declared invalid,
General rule: Implied amendment is never presumed
nothing has been amended. If the amendatory act
nor favoured. Every statute should be harmonized with
is complete by itself, it will be considered as an
other laws on the same subject.170
original or independent act.177 If the amendatory
Exception: A clear inconsistency and irreconcilable
act is declared
repugnancy between statutes.171
invalid, it would be as if the amendment did not
exist; the prior act remains unaffected
Effectivity
and in force.178
- Amended takes effect 15 days following the
publication in the Official Gazette or a newspaper
Revision and Codification
of general circulation172
- The purpose is to restate the existing laws into one
- An amended is a part of the original act that is statute, simplify complicated provisions, and make
already in force and effect, hence, it becomes laws on the subject easily found
effective as part of the amended law at the time
- It is a continuation of the existing statutes
the amendatory act takes effect General rule:
Presumption: The author has maintained a
Amended act operates prospectively Exceptions:
consistent philosophy or position. The code is
1. If the legislature clearly intended its retroactive enacted as a single, comprehensive statute, and is
effect to be considered as such and not as series of
2. No vested rights impaired disconnected articles or provisions.
3. Procedural laws are applicable to actions
pending and undetermined at the time of In cases of irreconcilable conflict
passage 1. Interpretation which is best in accord with the
general plan
Effects 2. Later in physical position, being the latest
Presumption: The legislature would not have amended expression of legislative will
it had it not wanted to change its meaning 3. What is omitted is deemed repealed because
- Meaning of law changes because an amended act the codification is intended to be a complete
should be given a different construction from that enactment on the subject. But this is limited
of the original. only if revision or codification is intended to be
- As if the original statute has been repealed and a a complete enactment on the subject,
new and independent act in the amended form expression of the whole law, or if it revises the
had been adopted whole subject matter of the former statute179
- As if the statute has been originally enacted in its 4. A change in phraseology does not
amended form automatically result to change in construction
- The provisions of the original act not affected by of the old laws, neither an alteration nor
the amendment remain in force, while those omission or addition of words in the later
omitted are deemed repealed173 statute shall be held to alter the construction
- Jurisdiction of a court is determined by the law in of the former acts. This is because in
force at the time the action is instituted. And once codifications, condensation is a necessity;
the court has acquired jurisdiction, that will remain general ideas will be expressed in brief phrases.
with it until the case is decided.174 o If a statute
amends a prior act with the effect of divesting the Mecano v. COA
court of jurisdiction, it may not Petitioner was hospitalized for cholecystitis. He filed for
be construed as to oust refund or reimbursement of the hospital expenses he
jurisdiction that may have incurred. COA denied his request since the Administrative

170
Estrada v. Caeda
175 Government v. Gale
171 Garcia Valdez v. Tuason
176 Erectors, Inc. v. NLRC
172 Art. 2 (CC), as amended by EO 200
177 People v. Lim
173 Parras v. Land Registration Commission
178 Government v. Agoncillo
174 Rillaroza v. Arciaga 179 Mecano v. COA
JUSTIN SUCGANG | BLOCK 2 ATTY. WIGBERTO TANADA, JR.
Code of 1987 repealed the RAC, and the same
section from which Mecano based his action has not been - But implied repeals are not favoured since the
reproduced. presumption is against inconsistency or
Held: The later code did not cover not attempted to cover
repugnancy
the entire subject matter. There are several matters treated
Presumption: The legislature knows the existing laws
in the old Code that are not found in the new one. The fact
that a later law may relate to the same subject matter is in the subject and could not have enacted
insufficient to cause an implied repeal since it may just be inconsistent or conflicting statutes. Exception: There
cumulative of the prior legislation. is an irreconcilable repugnancy

Repeal Iloilo Palay and Corn Planters Association v. Feliciano


- The power to enact a law necessarily includes the Respondent Manager of the Rice and Corn Administration
urged the president to import rice. Petitioners alleged that
power to repeal it for the legislature cannot enact
RA 3452 prohibited the importation of rice by the
an irrepealable laws to limit the future legislative government, only private parties may do so. The
acts government responded and said that RA 2207 allowed the
- Only the legislature can repeal laws. The Supreme President to import rice should there be shortage or national
Court can only promulgate rules of procedure and emergency. Held: RA 2207 covers three periods, and only
cannot exercise such power to alter, change or prohibits importation when there is sufficiency of rice supply
repeal substantive laws and normalcy of times. Ra 3452 covers only the first and
- Every legislative body may modify or abolish the second situation (i.e. shortage is not enough to constitute a
national emergency). Hence, the two laws can be
acts passed by itself or its predecessor and may be
harmonized. Ban on importation disappears when the
exercised at the same session which the original
country is confronted of rice shortage.
act was passed, and even where the bill is in its
progress and before it People v. Santayana
becomes a law Santayana was designated as a special agent of the CIS and
was issued a firearm. He was apprehended in Plaza Miranda
Total repeal is rendered revoked completely, while a and was charged for illegal possession of firearms since he
partial repeal leaves the unaffected portions of the had no permit to carry. CFI Manila tried and convicted him.
statute in force. It was alleged that the CFI Manila had no jurisdiction and the
Municipal Court should have tried him.
A declaration in a statute that a particular and specific Held: The two provisions may be harmonized by holding that
the CFI Manila and the Municipal Court have concurrent
law (identified by its number and title) is repealed is an
jurisdiction since the penalty that may be imposed both fall
express repeal. All other repeals are implied repeals.
under the said courts. Aside from this, the Macarandang
ruling was still prevailing at the time of his apprehension.
A law is repealed only by the enactment if subsequent The Mapa doctrine were it did not exempt special agents
laws. Violations, non-observance, disuse, customs and from permit to carry weapons were promulgated during the
practices on the contrary do not render a prior law pendency of Santayana’s case.
repealed.
C&C Commercial v. NAWASA
Repeal by implication RA 912 mandates that Filipino corporations should be
- Where a statute of a later date clearly reveals an prioritized. NAWASA awarded the bid to a foreign company
in constructing water pipes in several provinces. NAWASA
intention on the part of the legislature to abrogate
argues that it was not under the term “government”
a prior act
required to prioritize Filipino companies in projects as
- This intention must be clear and manifest - provided in RA 912. Held: CA 138 and RA 912 are statutes
Categories: o Two acts on the same subject in pari material and be construed together. The former
and have the same object are in an irreconcilable included purchases by government-owned companies (like
conflict, the later act to the extent of the NAWASA), while the later only talked about construction or
conflict constitutes an implied repeal of the repair work done by the government.
former
o The later act covers the whole Primicias v. Municipality of Urdaneta Primicias was
apprehended and charged for violation of a local ordinance
subject of
prohibiting overtaking. Held: The ordinance is invalid since
the earlier one and is intended RA 4136 explicitly repealed Act 3922. The ordinance
as a substitute violated several requirements of RA 4136, specifically the
Basis: A cardinal rule in science of jurisprudence that clearness, definiteness and certainty of the prohibitions.
two inconsistent laws on the same subject cannot No distinctions has been made between the vehicles
co-exist in one jurisdiction180 Maxim: leges covered, the thouroughfares, required under Sec. 38. RA
posteriors priores contrarias abrogant (the later 4136 specifically mentioned that no ordinances may be
statute repeals prior ones which are repugnant to it)

180 Garcia Valdez v. Tuason


JUSTIN SUCGANG | BLOCK 2 ATTY. WIGBERTO TANADA, JR.
passed specifying maximum allowable speed
other than those provided in it. 3. Other forms of repeal
- The most powerful is that which arises
- Hence, a general law does not repeal a special one. when the later law is expressed in the form
The special shall be considered as an extension to of a universal negative, since a negative
the general law. But, there is always a partial statute repeals all conflicting provisions
repeal where the later act is a special law. - Where it enacts something in general
Presumption: The legislature considers and makes terms and afterwards it passes another on
provisions for all circumstances of the particular case the same subject, which though expressed
Exception: Where the words used in a general law is so in affirmative language introduces special
broad and so clear and explicit as to show the intention condition
to cover the whole subject or restrictions

Villegas v. Subido
Effects of repeal
After the retirement of the Asst. City Treasurer, Civil Service
Commissioner Subido authorized Jose Gloria to assume the
1. Renders inoperative as of the date the
position. Mayor Villegas ordered Gloria to refrain from repealing act takes effect. It is a declaration
exercising his duties since the appointment power is lodged that the repealed statute is invalid from the
with him under the Decentralization Act. During the date of enactment
pendency of the case, the President nominated Gloria and 2. But it does not undo the consequences of the
was duly confirmed. operation of the statute while in force, unless
Held: The City Charter of Manil gave the President the power directly expressed by the language or necessary
to appoint the Asst. City Treasurer. The Revised implications. It cannot render illegal what was
Administrative Code (RAC) only allowed appointment of
legal before
employees by local officials, and not officers. The Charter is
considered as an exception to the general rule found in the 3. Cannot oust jurisdiction from a court, nor
RAC. render its decision void
4. Defeats all actions and proceedings still
CEPALCO v. Commissioner of Internal Revenue Petitioner pending which arose from the repealed statute
was granted a franchise in 1961 to maintain an electric light, since an appellate court will dispose of a
heat and power system in Cagayan de Oro. It was question according to the law
successively amended to include other municipalities. Later, prevailing at the time of rendition of the
the Local Tax Code was enacted empowering local appealed judgment
government units to impose new taxes. CEPALCO refused to
5. Does not destroy or impair vested rights under
pay the additional tax to the province of Misamis Oriental
the statute prior its repeal. Rights accrued and
claiming that its franchise expressly provided that it would
only pay 3% from its gross earnings.
vested while a statute is in force survives
Held: The Local Tax Code is more general in scope and repeal
application, while the franchise grant to CEPALCO is specific. 6. It will not affect the terms of contract entered
into by parties on the basis of the repealed law.
1. Implied repeal by revision or codification And will apply even if one of the contracting
- Legislative intent is shown by enactment of parties is the government.181
a statute revising or codifying the former 7. It does not preclude collection of taxes under
laws on the whole subject matter the repealed law
- Legislative declaration that whatever is 8. But will deprive the court of its jurisdiction if a
embraced in the new statute shall prevail penal law is repealed since the act is not
and whatever is excluded shall be anymore considered criminal
discarded 9. Repeal of a municipal charter destroys all
offices under it, and puts an end to the
2. Repeal by re-enactment functions of an incumbent, except those saved
- A re-enacted statute of the by the charter
whole subject matter, if 10. Simultaneous repeal and re-enactment does
complete and comprehensive is not affect the rights and liabilities accrued
regarded as the whole law on the subject. under the original statute since the re-
- When a specific section of a prior act by enactment simply neutralizes the repeal
providing that it should be read as follows, 11. A law which expressly repeals a prior law is
then quote the amended provisions, what repealed, the first law shall not be
is not included in the revived unless expressly stated
re-enactment is deemed repealed

181 Insular Government v. Frank


JUSTIN SUCGANG | BLOCK 2 ATTY. WIGBERTO TANADA, JR.
Tac-an v. CA Held: The signature/approval of the Governor
Tac-an caused the Acopiado brothers to thumb mark a deed is needed since the brothers were non-Christians and
of quit claim as payment for his services as lawyer. The Subanons. The Administrative Code of Mindanao and Sulu
brothers later dismissed his services since their relatives did was still in force at that time and was not repealed.
not want to give the land as payment. One of the brothers
sold his share of land, hence, Tac-an filed an action to annul Distinctions to expiration of law
the sale. He asked the governor to sign the deed. It was - Unlike in expiration of law, absolute repeal
signed but was later withdrawn for deceit and false
obliterates the crime and erases the stigma of
representation on his part.
conviction.

JUSTIN SUCGANG | BLOCK 2 ATTY. WIGBERTO TANADA, JR.

You might also like